You are on page 1of 43

BOARD QUESTIONS IN PHARMACOLOGY

CHOOSE THE BEST ANSWER:

MPL = 44.5

1. A 40 year old female who underwent total abdominal hysterectomy with bilateral
salpingo-oophorectomy. To prevent osteoporosis what HRT regimen will be best for her.
She claims to have no history of breast cancer in the family:
A. Estrogen only preparation
B. Estrogen and medroxyprogesterone acetate
C. Medroxyprogesterone acetate
D. Raloxifene
MPL- 0.25
REFERENCE: PHARMACOLOGY 4th Ed. Rang, pp. 440
2. The Patient at the delivery room is having uterine atony after giving birth to an 8-lb baby
boy.
Her BP at present is 150/90 mmHg. What agent should you give her:
A. Methylergonovine maleate
C. Carboprost
B. Oxytocin
D. Ergometrine
MPL: 1
REFERENCE: PHARMACOLOGY 4th Ed. Rang, et al
pp.450
3. An old female was treated for endometriosis for 6 months. She claims to have been
given a drug with androgenic effects such as hirsutism deepening of the voice and acne.
Which of the following drugs could have been given:
A. GnRH agonist
C. Danazol
B. Combined oral contraceptive pills
D. Medroxyprogesterone acetate
MPL: 0.25
REFERENCE: PHARMACOLOGY 4th Ed. Rang,
pp.447
4. A patient underwent hip replacement and is being given morphine for pain relief. At
present her RR was noted to be at 8 cycles/min with prolonged episodes of apnea. What
will you give in this patient?
A. Atropine sulfate
C. Naloxone
B. Flumazenil
D. Protamine sulfate
MPL; 1
REFERENCE: PHARMACOLOGY 4th Ed. Rang, pp.598
5. If you have a patient with seizure disorder which anesthetic agent will not be appropriate:
A. Enflurane
C. Halothane
B. Nitrous oxide
D. Procaine
MPL: 0.25
REFERENCE: PHARMACOLOGY 4th Ed. Rang, pp.523

6. A 50-year-old male with COPD is complaining of difficulty of sleeping for almost one
month. Which of the following sedative hypnotics would be best for him:
A. Diazepam
C. Zolpidem
B. Pentobarbital
D. Thiopental
MP: O.25
REFERENCE: PHARMACOLOGY 4th Ed. Rang,
pp.534
7. On waking up a patient who just underwent knee surgery cannot remember what
happened while he is at the Operating Room. The anxiolytic agent that was probably
given to him was _______.
A. Buspirone
C. Hydroxyzine
B. Lorazepam
D. Thiopental
MPL: 0.25
REFERENCE: PHARMACOLOGY 4th Ed. Rang,
pp.536
8. A 22-year-old female was found unconscious in her room. She presently broke up with
her boyfriend. By her bedside a number of sleeping pills and whiskey was noted.
What could be the possible effect and interaction that occurred?
A. Additive
C. Potentiation
B. Antagonism
D. Synergism
MPL: 0.33
9. If the patient will be given antidepressants. What adverse effect could be anticipated?
A. Insomnia
C. Sedation
B. Diarrhea
D.Tachycardia
MPL: 0.33
REFERENCE: PHARMACOLOGY 4th Ed. Rang, pp.556
10. Antipsychotic potency generally runs parallel to the activity on which of the following
receptors:
A. D1
B. D2
C. D3
D. D4
MPL: 0.25
REFERENCE: PHARMACOLOGY 4th Ed. Rang, pp.487
Next page pls.

11. The WHO classification of amphetamines:


A. Cognition enhancer
C. Neuroleptic
B. Hallucinogen
D. Psychostimulants
MPL: 0.33
REFERENCE: PHARMACOLOGY 4th Ed. Rang, pp.469
12. If the patient claims to have visual hallucinations, what could be the possible
psychotropic drug class that was taken?
A. Antidepressant
C. Psychotomimetic
B. antipsychotic
D. Psychomotor stimulant
MPL: 0.25
REFERENCE: PHARMACOLOGY 4th Ed. Rang, pp.469
13. A student would like to improve her memory and cognitive performance. What class of
psychotropic drug will she choose:
A. Cognitive enhancers
C. Psychostimulants
B. Psychotomimetics
D. Thymoleptics
MPL: 1
REFERENCE: PHARMACOLOGY 4th Ed. Rang, pp.469
14. A bone cancer patient took high doses of morphine to relieve pain. What agent can be
used to reverse its toxic effects?
A. Nalbuphine
C. Nalorphine
B. Naloxone
D. Methadone
MPL: 1
REFERENCE: PHARMACOLOGY 4th Ed. Rang,pp.598
15. If respiratory depression would be the marked effect of the opioid analgesic, it is
probably more selective to what receptor subtype?
A. Delta
C. Kappa
B. Gamma
D. Mu
MPL: 0.25
REFERENCE: PHARMACOLOGY 4th Ed. Rang,pp.592
16. Which of the following is not a manifestation of a patient taking an opioid analgesic.
Selectively interacting with mu receptors?
A. Analgesia
C. Physical dependence
B. Dysphoria
D. Respiratory depression
MPL: 0.25
REFERENCE: PHARMACOLOGY 4th Ed.
Rang,
pp.592
17. If a child was brought to the clinic with skin abscess. Wound cotton done revealed the
presence of Staph aureus. What antibiotic will be appropriate for this child:

A. Amoxicillin
C. Cloxacillin
B. Cephalexin
D. Vancomycin
MPL: 1
REFERENCE: PHARMACOLOGY 4th Ed.
Rang,
pp.693
18. The second generation cephalosporin which crosses the blood brain barrier:
A. Cefoxime
C. Cefoxitin
B. Cefuroxime
D. Cefoperazone
MPL: 1
REFERENCE: PHARMACOLOGY 4th Ed.
Rang,
p.694
19. A G2P1 term, will be given prophylactic antibiotic prior to CS. Which of the following
should be recommended:
A. Cephalexin
C. Cefazolin
B. Cefoxitin
D. Ceftriaxone
MPL: 0.33
20. After prolonged treatment with penicillin, the patient developed pseudomembranous
colitis. What antibiotic will you give for this case:
A. Aztreonam
C. Imipenem
B. Ceftriaxone
D. Vancomycin
MPL: 0.25
REFERENCE: PHARMACOLOGY 4th Ed. Rang,p.702
21. This agent inhibits topoisomerase II (DNA gyrase)
A. Clindamycin
C. Spectinomycin
B. Ciprofloxacin
D. Tetracycline
MPL: 0.25
REFERENCE: PHARMACOLOGY 4th Ed. Rang,
p.720

Next page pls.

22. In case of gram-negative septicemia if Ampicillin will be given with Gentamicin. What
will be the expected response?
A There will be better chances of resolution
B B. Similar effect with monotherapy
C. Disease resolution will be delayed
D. No response will be noted
MPL:1
REFERENCE: PHARMACOLOGY 4th Ed. Rang Pp.
698
23.. A neonate was given IV antibiotics for 3 days. However, the baby had hypothermia,
diarrhea and other gray color. The baby was probably given:
A. Amikacin
C. Chloramphenicol
B. Erythromycin
D. Gentamycin
MPL: 1
REFERENCE: PHARMACOLOGY 4th Ed.
Rang,p.692
24. The following antibiotics inhibit protein synthesis. Which of the following inhibits the
translocation process:
A. Erythromycin
C. Netilmicin
B. Chloramphenicol
D. Tetracycline
MPL: 0.25
REFERENCE: PHARMACOLOGY 4th Ed.
Rang,p.699
25. The purpose of giving compound drug therapy in the treatment of tuberculosis:
A. to rapidly eradicate the strains of tubercle bacilli
B. to shorten the infections phase
C. to decrease the emergence of resistant organisms
D. to prevent complications of tuberculosis
MPL: 0.33
REFERENCE: PHARMACOLOGY 4th Ed.
Rang,p.706
26. The patient on the 4th month of anti-TB treatment is having peripheral neuropathy. This
may be attributed to which of the following:
A. Isoniazid
C. Pyrazinamide
B. Ethambutol
D. Streptomycin
MPL: 1
REFERENCE: PHARMACOLOGY 4th Ed.
Rang,p.704

27. Which of the following is NOT a first line agent in the treatment of tuberculosis?
A. Ethambutol
C. Rifampicin
B. Pyrazinamide
D. Streptomycin
MPL: 1
REFERENCE:

PHARMACOLOGY

4th

Ed.

Rang,p.706

28. This agent is NOT included in the management if lepromatous leprosy:


A. Capreomycin
C. Dapsone
B. Clofazimine
D. Rifampicin
MPL: 0.33
REFERENCE: PHARMACOLOGY 4th Ed.
Rang,p.706
29. It is important for the clinician to avoid selection of an antiretroviral regimen that
contains agents with similar toxicity profiles. Which regimen below contains at least
2 agents with similar toxicity profiles?
A. Zidovudine, Lamivudine, Indinavir
C. Zidovudine, Didanosine, Saquinavir
B. Didanosine, Zalcitabine, Nelfinavir
D. Stavudine, Lamivudine, Indinavir
MPL: 0.25
REFERENCE: PHARMACOLOGY 4th Ed.
Rang,p.714
30. A guanosine derivative which selectively inhibits viral DNA polymerase:
A. Aciclovir
C. Vidarabine
B. Amantadine
D. Zidovudine
MPL: 0.25
REFERENCE: PHARMACOLOGY 4th Ed.
Rang,p.713
31. A patient taking Warfarin was also being treated for fungal infection. After 7 days, the
patient developed epistaxis, Which of the following antifungal agents may have this
interaction with Warfarin?
A. Nystatin
C. Flucytosine
B. Ketoconazole
D. Amphotericin B
MPL: O.25
REFERENCE: PHARMACOLOGY 4th Ed.
Rang,p.317
Next page pls.

32. If the patient became anemia with neutropenia and thrombocytopenia during antifungal
treatment. . What is the possible agent she is taking?
A. Griseofulvin
C. Flucytosine
B. Fluconazole
D. Terbinafine
MPL: 0.25
REFERENCE: PHARMACOLOGY 4th Ed.
Rang,p.722
33. A7 year old patient was admitted because of pneumonia. On routine stool exam E.
Histolytic Cyst 5 8/hpf was noted:
A. Anti- amebic treatment not necessary
B. Give diloxanide furoate
C. Give metronidazole
D. Give both diloxanide furoate and metronidazole
MPL: 0.33
REFERENCE: PHARMACOLOGY 4th Ed.
Rang,p.736
34. Which of the following anti-malarial agent may promote radical cure?
A. Chloroquine
C. Primaquine
B. Mefloquine
D. Pyrimethamine
MPL: 0.25
REFERENCE: PHARMACOLOGY 4th Ed.
Rang,p.728
35. The drug of choice for mixed round worm infection:
A. Pyrantel pamoate
C. Niclosamide
B. Mebendazole
D. Praziquantel
MPL: 0.50
REFERENCE: PHARMACOLOGY 4th Ed.
Rang,p.740
36. On follow-up, after treatment with tapeworm infection, scolex was noted in the stool of
the child:
A. Praziquantel
C. Mebendazole

B. Niclosamide
D. Ivermectin
MPL: 0.25
REFERENCE: PHARMACOLOGY 4th Ed.
Rang,p.742
37. Classified as anti-cancer antimetabolite agent:
A. cyclophosphamide
C. Methotrexate
B. Doxorubicin
D. Paclitaxel
MPL: 0.25
REFERENCE: PHARMACOLOGY 4th Ed.
Rang,p.676
38. An agent used for cancer chemotherapy that inhibits purine synthesis:
A. Cytarabine
C. Mercaptopurine
B. Fluorouracil
D. Vincristine
MPL: 0.50
REFERENCE: PHARMACOLOGY 4th Ed.
Rang,p.677
39. Does not cause myelosuppression:
A. Bleomycin
C. Etoposide
B. Cisplatin
D. Dactinomycin
MPL: 0.25
REFERENCE: PHARMACOLOGY 4th Ed.
Rang,p.678
40. In a prescription made by an internist, the generic name was enclosed in a parenthesis
and written below the brand name. This will be interpreted by the drug store as:
A. Violate prescription
C. Erroneous prescription
B. Impossible prescription
D. Correct prescription
MPL: 0.50
Generics Act of the Philippines
41. If the physician makes a prescription order that utilizes a drug supplied by the
pharmaceutical company, what class of prescription order was made:
A. Extemporaneous
C. Precompounded
B. Compounded
D. Erroneous
MPL: 0.25
42. The response of a patient who has bronchial asthma on the following drugs maybe
graded as follows:
A. Epinephrine > Norepinephrine > Isoproterenol
B. Isoproterenol > Epinephrine >> Norepinephrine
C. Isoproterenol > Epinephrine = Norepinephrine
C
D. Isoproterenol = Epinephrine >> Norepinephrine

MPL: 0.33
Rang,p.160

REFERENCE: PHARMACOLOGY 4th Ed.

Next page pls.

-543. A patient with septic shock was noted to have absent urine output for the past 12 hours.
Which of the following drugs would be most helpful to this patient?
A. Furosemide
C. Dopamine
B. Norepinephrine
D. Epinephrine
MPL: 0.33
REFERENCE: PHARMACOLOGY 4th Ed.
Rang,p.288
44. A 40 year old male was seen at the ER because of anaphylaxis and was immediately
given Epinephrine. Apparently, The patient has taken prazosin for his hypertension.
Which of the following maybe observed in this patient?
A. The patient may develop hypertensive crisis.
a.
B. There will be no effect on the patients blood
pressure.
C. The patient may develop hypotension.
D
D. The patient may develop severe difficulty of breathing due to
bronchoconstriction.
E MPL: 0.33
REFERENCE: PHARMACOLOGY 4th Ed.
Rang,P.152

45. Which of the following actions of norepinephrine is blocked by prazosin?


A. Bronchial smooth muscle relaxation
B. Vascular smooth muscle contraction
B. Renin release
C. Increased heart rate
MPL: 0.25
REFERENCE: PHARMACOLOGY 4th Ed.
Rang,p.152
46. Bradycardia maybe noted as reflex response when a patient is given
A. Dobutamine
C. Norepinephrine
B. Propranolol
D. Methyldopa
MPL: 0.25
REFERENCE: PHARMACOLOGY 4th Ed.
Rang,p.127
47. Rifampicin and INH are given together for the treatment of pulmonary tuberculosis
because their drug interaction is
A. Additive
C. Potentiation
B. Synergism
D. Antagonism
MPL: 0.33
REFERENCE: PHARMACOLOGY 4th Ed.
Rang,p.703
48. If the plasma concentration of a drug declines with first order kinetic, this means that
A. there is only one metabolic path for drug disposal
B. the rate of elimination is proportional to the remaining concentration
b.
C. the drug is largely metabolized in the liver after
oral absorption and has low bioavailability
D. the drug is not distributed outside the vascular systems
MPL: 0.50
REFERENCE: PHARMACOLOGY 4th Ed.
Rang,p.87
49. The following is NOT phase II drug metabolized reaction:
A. Oxidation
C. Methylation
B. Glucoronidation
D. Acetylation
MPL: 0.25
REFERENCE: PHARMACOLOGY 4th Ed.
Rang,p.79
50. Mr. Jose Vida was admitted due to severe pneumonia. He was given a drug with a
volume of distribution of 40L and clearance of 80ml/min. What should be the loading
dose in order to achieve a therapeutic plasma concentration of 40mg/L?
A. 0.1 mg/L
C. 115.2 mg

B. 10 mg
D. 160 mg
MPL: 0.25
REF. BASIC & CLINICAL PHARMACOLOGY 9th Ed.
KATZUNGp.46
51. Two drugs A and B have the same mechanism of action. Drug A at a dose of 5 mg
produce the same magnitude of effect as drug B at a dose of 500 mg. This means that
A. Drug B is less efficacious than drug A
B. Drug A is 100x more potent than drug B
C. toxicity of drug A is less than that of drug B
F
D. Drug A is more effective than drug B.
MPL: 0.50
REFERENCE: PHARMACOLOGY 4th Ed.
Rang,p.29

Next page pls.

-6A 67 year old male was seen at the OPD due to epistaxis which occurred thrice for
the past
week and easy bruising. He also complained of abdominal discomfort and
lack of appetite.
PMH: Atrial fibrillation, Nocturnal heartburn
Social history: Chronic alcohol abuse
Medications: Warfarin, Digoxin, Cimetidine, Procainamide
Pertinent laboratory exam: INR 4.5

52. Which of the following could have contributed to the INR result of the patient?
A. Cimetidine
C. Chronic alcoholism
B. Procainamide
D. Digoxin
MPL: 0.50
REFERENCE: PHARMACOLOGY 4th Ed.
Rang,p.374
53. The INR result is brought about by:
A. Increased thrombin activity
B. Decreased platelet activation
C. Diminished levels of vitamin K
c.
D. Decreased levels of factors II, VII. IX and X
MPL: 1
REFERENCE: PHARMACOLOGY 4th Ed.
Rang,p.316
54. In the event this patient develop massive bleeding, which of the following is the best
thing to be done?
A. Discontinue the drug and give vitamin K
B. Discontinue the drug and change to LMWH.
d.
C. Discontinue the drug and give protamine
sulfate
e.
D. Discontinue the drug and transfuse platelets.
MPL: 0.33
REFERENCE: PHARMACOLOGY 4th Ed.
Rang,p.319
55. If this patients ventricular rate cannot be controlled with digoxin, Which of the
following maybe used as an alternative for ventricular rate control?
A. Amlodipine
C. Quinidine
B. Atenolol
D. Ibutilide
MPL: 0.25
REFERENCE: PHARMACOLOGY 4th Ed.
Rang,p.269
56. Which of the following is an adverse effect of procainamide?
A. Hypertension
C. drug induced Lupus erythematosus
B. Bradycardia
D. Corneal microdeposits
MPL: 0.25
REFERENCE: PHARMACOLOGY 4th Ed. Rang
p.229

A 26 year old medical student was brought to the ER for an apparent suicide poisoning
due to broken heart. The maid could not recall the medication but claimed that the bottle
contained medicine for headache. On PE, the physician noted cyanosis of the oral mucosa,
tongue and nail beds.
57. Which of the following is the most likely drug ingested?
A. Aspirin
C. Flurbiprofen
B. Acetaminophen
D. Meloxicam
MPL: 0.25
58. This DMARD agent is also an immunosuppressant.
A. Gold
C. Chloroquine
B. Penicillamine
D. Cyclosporine
MPL: 0.50
REFERENCE: PHARMACOLOGY 4th Ed. Rang
p.237
59. A 52 year old woman was suffering from severe joint pains and was diagnosed to have
RA. However, she also appears to be suffering from acid peptic disease. Among the
following, which is the safest to give her?
A. Aspirin
C. Celecoxib
B. Ibuprofen
D. Phenylbutazone
MPL: 0.50
Next page pls.

-760. Which of the following NSAID has the longest half-life and therefore should be given
at a longer dosage interval?
A. meloxicam (20h)
C. Tenoxicam (72h)

B. Rofecoxib (17h)
MPL: 1

D. Piroxicam (50-60h)
REFERENCE: PHARMACOLOGY 4th Ed. Rang p.230

A 21 year old male after quarrelling with his girlfriend took a bottle of insecticide from
their garden approximately 30 40 cc. He went back to his room and talked with his
girlfriend over the phone. While talking to his girlfriend, he complained of dizziness,
blurring of vision, generalized body weakness, numbness, severe epigastric pain and
shortness of breath. He was able to call the attention of his older sister before falling on the
floor. He was found lying on the floor and was noted to be pale, drowsy, with stiffening of
the trunk and extremities and soaked with sweat. He was then brought to the ER. On
arrival, he has drooling of saliva with 3 episodes of white viscid material, non-projectile
non-bile stained about 2 tbsp. per bout.
61. As an ER physician, which of the following are you going to give the patient?
A. Pilocarpine
C. Atropine
B. Physostigmine
D. Carbachol
MPL: 0.33
REFERENCE: PHARMACOLOGY 4th Ed. Rang p.122
62. A diabetic patient develops hyperlipedemia (elevated LDL, and triglycerides, normal
HDL). Which of the following is NOT appropriate for the patient?
A. Colestipol
C. lovastatin
B. Gemfibrozil
D. niacin
MPL: 0.25
REFERENCE: PHARMACOLOGY 4th Ed. Rang p.308
63 . The patient was noted to have an elevated triglycerides level after treatment with a
hypolipidemic agent. This side effect could be secondary to which of the following?
A. Cholestyramine
C. cerivastatin
B. Clofibrate
D. niacin
MPL: 0.25
REFERENCE: PHARMACOLOGY 4th Ed. Rang
p.307
64. Inhibits de novo synthesis of cholesterol by inhibiting HMG CoA reductase:
A. Atorvastatin
C. gemfibrozil
B. Colestipol
D. niacin
MPL: 0.50
REFERENCE: PHARMACOLOGY 4th Ed. Rang
p.305

65. The patient has been having intermittent episodes of dry cough. The agent that may have
probably caused this is ______________:
A. Captopril
C. losartan
f.
B. candesartan
D. telmisartan
MPL: 1
REFERENCE: PHARMACOLOGY 4th Ed. Rang
p.292
66. A patient with tachyarrhythmia and hypertension would be given an antihypertensive.
What agent would be the choice for this patient?
A. Felodipine
C. nifedipine
B. isradipine
D. verapamil
MPL: 0.25
REFERENCE: PHARMACOLOGY 4th Ed. Rang
p.275
67. -blockers are given with vasodilators due to the following reasons:
A. reduce renal sodium excretion
B. reduce renin release
C. decrease systemic vascular resistance
g.
D. increase cardiac contractility
MPL: 0.50
REFERENCE: PHARMACOLOGY 4th Ed. Rang
p.287
68. Which of the following conditions will NOT benefit from the use of VASODILATORS:
A. hypertension
C. angina
B. migraine
D. peripheral vascular disease
MPL: 0.50
REFERENCE: PHARMACOLOGY 4th Ed. Rang
p.290
Next page pls.

-8-

69. .An asthmatic patient came to the ER because of generalized wheezing in the lungs.
Urgent management was done. However after awhile hypertension was noted. Which
of the following medications was probably given?
A. salbutamol
C. terbutaline
B. isoproterenol
D. salmeterol
MPL: 0.50
REFERENCE: PHARMACOLOGY 4th Ed.
Rang P.160
70. .A flight attendant has been having allergic rhinitis. She comes to your clinic asking for
medication prescription 2 hours prior her to flight. What will be appropriate for her?
A. diphenhydramine
C. loratadine
B. Chlorpheniramine
D. promethazine
MPL: 0.50
REFERENCE: PHARMACOLOGY 4th Ed.
Rang p.345
71. It acts by inhibiting the immediate phase of asthma as it prevents mediator release from
mast cells
A. cromolyn sodium
C. montelukast
B. beclomethasone
D. zileuton
MPL: 0.33
REFERENCE: PHARMACOLOGY 4th Ed.
Rang p.347
72. An asthmatic patient maintained on oral salbutamol for 8 months, apparently had
infrequent asthma attacks for the first 6 months. However, at present she had increased
frequency of asthma attacks. This could be secondary to:
A. inadequate dose
C. hypoxemia
B. poor compliance
D. tachyphylaxis
MPL: 0.33
REFERENCE: PHARMACOLOGY 4th Ed.
Rang p.16
73. Which of the following is NOT a pharmacologic action of Cortisone:
A. anti-inflammatory
C. promotes fetal lung maturation
B. immunosuppressant
D. adrenal suppression
MPL: 0.33
REFERENCE: PHARMACOLOGY 4th Ed.
Rang p.416
74. A patient with severe persistent asthma was treated for 6 months now with prednisone.
Which of the following would be the expected side effect?
A. Cushingoid facie
C. hyperkalemia

B. hypoglycemia
D. hypotension
MPL: 0.50
REFERENCE: PHARMACOLOGY 4th Ed.
Rang p.423
75. If a patient after prolonged steroid treatment developed depression. What synthetic
corticosteroid agent is he taking?
A. cortisone
C. fludrocortisone
B. dexamethasone
D. prednisone
MPL: 0.50
REFERENCE: PHARMACOLOGY 4th Ed.
Rang p.552
FM, 40 year old male was seen at the clinic for check-up. He has bradykinesia, muscle
rigidity, pill rolling movement, resting tremors and shuffling gait.
76. The single most effective agent in the treatment og this disorder is
A. Levodopa
C. Carbamazepine
B. Baclofen
D. Tacrine
MPL: 1
REFERENCE: PHARMACOLOGY 4th Ed.
Rang p.509
77. Which of the following is NOT TRUE about this drug?
A. In clinical practice, it is almost always used in combination with peripherally
acting inhibitor of aromatic L-amino acid decarboxylase.
B. This drug has a short half-life about 1 3 hr.
C. Administration of this drug with meals delay its absorption.
h.
D. This drug has very little side effect and can be
withdrawn abruptly.
MPL: 0.33
REFERENCE: PHARMACOLOGY 4th Ed.
Rang p.510
78. Which of the following is a selective inhibitor of MAO-B
A. Selegiline
C. Tolcapone
B. Amantadine
D. Bromocriptine
MPL; 0.50
REFERENCE: PHARMACOLOGY 4th Ed.
Rang p.511

Next page pls.

-9GM, a 53 year old male was seen at the OPD for regular check-up. He had a history of
type II DM for 5 years and hypothyroidism for 20 years.
Medications: Levothyroxine, Glyburide
79.The patient had a series of laboratory examinations and was noted to have
hyperlipedemia. Which of the following drugs can interfere with the absorption of
levothyroxine?
A. Simvastatin
C. Gemfibrozil
B. Cholestyramine
D. Niacin
MPL: 0.33
REFERENCE: PHARMACOLOGY 4th Ed.
Rang p.70
LC, a 25 year old patient was seen at the clinic because of lump on her throat. She
has gained 5 kg for the past 6 months, has cold intolerance, unusual painful heavy menses
and constipation. She complained that the lump on her throat interferes with her
swallowing.
PMH: iron deficiency anemia of 4 months duration
Medications: Ferrous sulfate, Ibuprofen, Kelp tablets(contains Iodine), Lo-ovral
(Ethinyl estradiol and norgestrel) daily
80.What effect does the use of oral contraceptive have on the patients hypothyroidism?
A. Low dose oral contraceptives are unlikely to affect her test results.
B. Estrogen may falsely increase TSH due to alterations in TSH secretion
C. Estrogen may falsely decrease FT4I due to alterations in TBG.
D. Estrogen may falsely elevate total T4 levels due to alterations in TBG.

MPL: 0.33
REF. BASIC & CLINICAL PHARMACOLOGY 9th Ed.
KATZUNGp.630
81. If LC becomes pregnant, how might her levothyroxine replacement be affected?
A. She may require a 20-30% increase in dose.
B. She may require a 20-30% decrease in dose.
C. She will probably not require any changes in dose.
D. TT4 levels should also be monitored because of changes in TBG.
MPL: 0.33
REF. BASIC & CLINICAL PHARMACOLOGY 9th Ed. KATZUNG
p.631
A 62 year old male was seen at the ER because of lightheadedness, palpitation and
shortness of breath. He said that the palpitations were associated with exercise that usually
went away with rest. Two days ago, while washing the dishes, he began to have shortness
of breath and felt that his heart was racing.
PMH: hypertension x 20 years; Hyperlipidemia for 5 years, and RHD with MVP
as a child.
Medications: Lisinopril, Furosemide, Gemfibrozil
82. Which of the following drugs is likely to be least effective in controlling this patients
rapid ventricular response?
A Digoxin
C. Verapamil
B. Diltiazem
D. Atenolol
MPL: 0.25
REFERENCE: PHARMACOLOGY 4th Ed. Rang
p.265
83. This patient was initially given verapamil for his AF. Due to the addition of this drug,
which of the following is the most appropriate treatment of this patients hypertension?
A.Continue lisinopril and furosemide
B. Continue lisinopril and discontinue furosemide
C. Continue lisinopril
D. Continue lisinopril, discontinue furosemide and add hydrochlorothiazide
MPL: 0.25
REFERENCE: PHARMACOLOGY 4th Ed. Rang
p.361
A 37 year old Fil-Am was seen at the clinic because of nausea and vomiting. Five
days prior to consultation, he had nausea, vomiting, fever and chills. He took ibuprofen for
generalized ache 3 days ago. Recent blood sugar reading was unavailable because he ran

out of test strips . On admission, he was weak looking, severely dehydrated with weak
pulses.
PMH: DM type I for 21 years, one episode of DKA 4 months ago; peripheral
neuropathy
Medications: Insulin NPH< Ibuprofen, Amitryptilline
Next page pls.

- 10 84. Which of the following agents is least likely to aggravate the patients condition?
A. Furosemide
C. Acetaminophen
B. Amitryptilline
D. Insulin
MPL: 0.33
REFERENCE: PHARMACOLOGY 4th Ed. Rang
p.363
85. Loop diuretics have their principal diuretic effect on:
A. collecting ducts
B. ascending limb*
C. distal convoluted tubules
D. proximal convoluted tubules
MPL: 0.25
REFERENCE: PHARMACOLOGY 4th Ed. Rang
p.361
86. Which of the following are potential side effects of thiazide diuretics?
A. hypokalemia, hyperglycemia, hyperlipidemia
B. hypokalemia, ototoxicity, hyperuricemia
C. hyperkalemia, alkalosis, nausea/vomiting
D. hyperglycemia, hypokalemia, metabolic alkalosis*
MPL: 0.33
REFERENCE: PHARMACOLOGY 4th Ed. Rang
p.364

87. Patients receiving insulin therapy generally require which of the following interventions
with progression to end-stage renal disease?
A. Decrease in total insulin dose
C. Increase in frequency of insulin
administration
B. Increase in total insulin dose
D. No change in insulin regimen
MPL: 0.25
REF. BASIC & CLINICAL PHARMACOLOGY 9th Ed. KATZUNG
p.694
BK a 23 year old female seen at the ER because of nausea and vomiting. Myalgia,
polydipsia and polyuria. Three days prior to consult, he attended a party and drank an
excessive amount of alcohol. He woke up sick to his stomach, vomited 6x since then and
was unable to eat nor drink. He stopped taking insulin and currently has headache.
PMH: Type I DM for 11 years. Depression and allergic rhinitis
Medications: Human insulin, Sertraline, Fluticasone, Loratidine, Acetaminophen

a.

B. blocking the release of Ach from nerve endings


D. stimulating vagus
REFERENCE: PHARMACOLOGY 4th Ed. Rang

MPL; 0.25
p.130
91. A 60 year old male underwent cataract removal. After surgery Ach chloride was
administered intraocularly to:
A. Relax the circular muscle of the iris
C. decrease tearing from lachrymal
secretion
b.
B. ensure complete miosis
D. decrease the flow of aqueous humor
MPL; 0.50
REFERENCE: PHARMACOLOGY 4th Ed. Rang
p.118

Next page pls.


88. If Bk.s blood glucose does not decrease by at least 2.8 mg/dl in the first 2 hours, what
should be done?
A. Double insulin infusion rate
C. Increase the insulin infusion to
0.5 /kg/hr
B. Give insulin bolus dose of 2U/kg
D. Check blood glucose again in 2
hours
MPL: 0.50
89. Which of the following statements about mixing NPH and regular insulins is NOT
TRUE?
A. The injection must be taken within 10 mins of mixing
B. Regular insulin is drawn up before the NPH dose
C. Mixtures of NPH and regular insulin are stable in any ratio
D. Mixing insulins allows greater flexibility versus 70/30 insulin
MPL: 0.25
90. Clostridium botulinum toxin produces respiratory paralysis by
A. blocking nicotinic receptors
C. causing circulatory
collapse

- 11 92. An elderly patient was diagnosed to have open angle glaucoma. She was given 0.25%
pilocarpine 2 gtts every 6 hours. The anticipated effect after the administration would
be
A. relax ciliary muscle
B. improve accommodation
C. relax sphincter muscle of iris
D. contract ciliary muscle and pull on trabecular network to relieve pressure

MPL: 0.33
REFERENCE: PHARMACOLOGY 4th Ed. Rang
p.121
93. Tachyphylaxis maybe observed with the use of the following:
A. Propranolol
C. Ephedrine
B. Alpha methyl tyrosine
D. Phenylephrine
MPL: 0.50
REFERENCE: PHARMACOLOGY 4th Ed. Rang
pp.16, 158
94. A 25 year old patient weighing 180 lbs consulted at the clinic. She tried several
methods of losing weight but failed. Her height is 170 cm. Based on her BMI she is
considered:
A. healthy
C. Obese
B. overweight
D. underweight
MPL: 0.33
REFERENCE: PHARMACOLOGY 4th Ed. Rang
p.399
95. This inhibitor of GIT lipase is useful in the treatment of obesity and has minimal side
effects:
A. sibutramine
C. Orlistat
B. Phentermine
D. glucosan
MPL: 0.33
REFERENCE: PHARMACOLOGY 4th Ed. Rang
p.407
96. A group of students studied the effects of various GI drugs by measuring the effects on
gastric pH and volume. The baseline gastric volume was 2cc and when tested with
litmus paper, the paper remained pink. After administering drug A, the gastric volume
was 2 cc and the litmus paper turned blue. This means
A. The drug is effective in lowering gastric Ph
c.
B. The drug is effective in increasing gastric pH
but no effect on the volume
d.
C. The drug is effective in lowering the gastric pH
and volume
e.
D. The drug has no effect at all on volume and ph
of gastric juice.
MPL: 0.50
REFERENCE: PHARMACOLOGY 4th Ed. Rang
p.375
97. A patient taking this drug came into the clinic due to visual disturbances, constipation,
difficulty in urination and dry mouth. This drug could be:

A. muscarinic antagonist
C. dopamine D2 antagonist
B. histamine H2 antagonist
D. gastrin antagonist
MPL: 0.50
REFERENCE: PHARMACOLOGY 4th Ed. Rang
p.122
98. The onset and duration of action of NPH insulin are extended because:
A. protamine decreases the rate at which insulin is absorbed
B. protamine blocks insulin metabolism in the liver
C. protamine is basic and combines with insulin by charge interactions
f.
D. protamine is slowly degraded proteolytically
releasing the bound insulin
MPL: 0.25
REF. BASIC & CLINICAL PHARMACOLOGY 9th Ed. KATZUNG
p.697
99. Which of the following is NOT TRUE about insulin action?
A. It stimulates glycogen synthesis in muscle fiber
B. It inhibits lipolysis in the adipocyte
C. It stimulates K fatty acid synthesis in the hepatocytes
g.
D. It stimulates gluconeogenesis in the
hepatocytes
MPL: 0.50
REFERENCE: PHARMACOLOGY 4th Ed. Rang
p.387
100. Which of the following drugs is most likely to improve gastric emptying in-patients
with diabetic gastroparesis?
A. Loperamide
C. magnesium hydroxide
B. Cisapride
D. Sucralfate
MPL: 0.50
REFERENCE: PHARMACOLOGY 4th Ed. Rang
p.381
End

Questions

Answer
A

1. Extrapyramidal adverse effect is commonly seen with the


use of this antipsychotic agent:
A. Haloperidol
C. Chlorpromazine
B. Thioridazine
D. Fluoxetine
2. An anti-malarial agent which also exerts an amebicidal
effect
A. Chloroquine
C. Primaquine
B. Chloroguanide
D. Mefloquine
3. An anti-arrhythmic drug that predominantly block the slow
calcium channels
A. Verapamil
C. Lidocaine
B. Amiodarone
D. Metoprolol
4. Repeated exposure to which of the following anesthetic
agents is most associated with liver damage
A. Isoflurane
C. Nitrous oxide
B. Halothane
D. Enflurane
5. The pharmacokinetic parameter which determines the
speed of drug input that must balance the speed of drug
elimination to achieve a steady state concentration
A. clearance
C. dosing rate
B. bioavailability
D. volume of distribution

6. If a drug has a high distribution volume, it indicates that:


A. it is highly bound to plasma proteins
B. drug is highly retained in the vascular compartment
C. the plasma drug concentration will increase
D. there is extensive tissue uptake

7. In what condition are the drugs phenoxybenzamine and


phentolamine useful?
A. pheochromocytoma
C. essential hypertension

Reference
Goodman
and Gilman
10th ed
p.488

MPL
1.0

Goodman
and Gilman
10th ed
p.1084
Goodman
and Gilman
10th ed
p.949
Goodman
and Gilman
10th ed
p.351-352
Basic and
Clinical
Pharmacol
ogy 9th Ed.
Katzung
p.34-41
Basic and
Clinical
Pharmacol
ogy 9th Ed.
Katzung
p.35 & 47
Basic and
Clinical
Pharmacol

1.0

0.25

0.33

0.50

0.25

1.0

B. migraine attacks

D. benign prostatic hyperplasia

8. A college friend consulted you regarding the suitability of


the therapy his doctor prescribed for severe
hypertension. He complained of postural and exercise
hypotension (dizziness), diarrhea and problems with
ejaculation. Which of the following is most likely to
produce the effects that your friend has described?
A. Propanolol
C. Hydralazine
B. Guanethidine
D. Captopril
9. Norpepinephrine acts as a neurotransmitter at:
A. all preganglionic neurons
B. parasympathetic postganglionic neurons
C. sympathetic postganglionic neurons
D. neuromuscular junction

10. In the treatment of organophosphate poisoning, the


mechanism of action of pralidoxime is:
A. competitive inhibition with acetylcholine
B. non-competitive antagonism with acetylcholine
C. regeneration of acetylcholine
D. regeneration of acetylcholinesterase
11. A patient was given digitalis for heart failure, complained
of nausea, vomiting, and diarrhea. PE revealed
irregular cardiac rhythm. Which of the following is the
most appropriate thing to do?
A. increase the dose of digitalis
B. add a beta-blocker
C. discontinue digitalis
D. administer potassium
12. A 52 year old man consulted at the clinic due to
hypertension (BP 190/90). Except for mild wheezing,
his PE findings were unremarkable. The drug given can

ogy 9th Ed.


Katzung
p.144-146
Basic and
Clinical
Pharmacol
ogy 9th Ed.
Katzung
p.166

0.33

Basic and
Clinical
Pharmacol
ogy 9th Ed.
Katzung
p.145
Basic and
Clinical
Pharmacol
ogy 9th Ed.
Katzung
p.103
Basic and
Clinical
Pharmacol
ogy 9th Ed.
Katzung
p.993

0.33

Basic and
Clinical
Pharmacol

0.33

0.33

0.25

depress AV conduction, hence the use of beta-blocker


is contraindicated. The drug that was given is:
A. Methyldopa
C. Verapamil
B. Enalapril
D. Hydrochlorothiazide
13. A patient noted to have elevated cholesterol, LDL, and
VLDL. Which among the following drugs act by
blocking HMG-CoA reductase to inhibit cholesterol
synthesis?
A. Probucol
C. Lovastatin
B. Clofibrate
D. Nicotinic acid
14. A 20 year old female was treated for endometriosis for 6
months. She claims to have been given a drug with
androgenic effects such as hirsutism, deepening of the
voice, and acne. Which of the following drugs could
have been given?
A. GnRH agonist
B. Combined oral contraceptive pills
C. Danazol
D. Medroxyprogesterone acetate
15. A patient underwent hip replacement and was given
morphine for pain relief. At present her RR was noted
to be at 8 cycles/min with prolonged episodes of
apnea. What will you give this patient to reverse the
present condition?
A. atropine sulfate
C. Naloxone
B. Flumazenil
D. Protamine sulfate
16. A 7-year old was admitted because of pneumonia. On
routine stool exam E. histolytica cyst 5-8/hpf was
noted. What will you give?
A. Diloxanide furoate
C. both A& B
B. metronidazole
D. no treatment necessary
17. A patient with septic shock was noted to have absent
urine output for the past 12 hours. Which of the
following drugs would be most helpful to this patient?
A. Furosemide
C. Dopamine
B. Norpeinephrine
D. Epinephrine
18. A 40 year old male was seen at the ER because of
anaphylaxis immediately given epinephrine.
Apparently, the patient has taken prazosin for his

ogy 9th Ed.


Katzung
p.235-236
C

Goodman
and Gilman
8th ed
p.881-884

1.0

Pharmacology 4th ed
page 447

0.25

Pharmacology 4th ed
Rang
Page 598

1.0

Pharmacology 4th ed
Rang
Page 736

0.33

Pharmacology 4th ed
Rang
Page 288

0.33

Pharmacology 4th ed
Rang

0.33

hypertension. Which of the following may be observed


in this patient?
A. He may develop hypertensive crisis.
B. There will be no effect on his blood pressure
C. He may develop hypotension.
D. he may develop severe difficulty of breathing due to
bronchoconstriction
19. Two drugs A and B have the same mechanism of action.
Drug A at a dose of 5 mg produce the same magnitude
of effect as drug B at a dose of 500 mg. This means
that
A. drug B is less efficacious than drug A
B. drug A is 100x more potent than drug B
C. toxicity of drug A is less than that of drug B
D. drug A is more effective than drug B
20. A diabetic patient developed hyperlipedemia (elevated
LDL, and triglycerides, normal HDL). Which of the
following is least appropriate for the patient?
A. Colestipol
C. Lovastatin
B. Gemfibrozil
D. Niacin
21. A patient with tachyarrhythmia and hypertension would be
given an antihypertensive. What agent would be the
best choice for this patient?
A. Felodipine
C. Nifedipine
B. Isradipine
D. Verapamil
22. Which of the following conditions will NOT benefit from the
use of vasodilators?
A. hypertension
C. angina
B. migraine
D. peripheral vascular disease
23. A patient at the delivery room is having uterine atony
after giving birth toan 8-lb baby boy. Her BP at present
is 150/90 mmHg. What agent should you give her:
A. methyergonovine maleate
B. oxytocin
C. carboprost
D. ergometrine
24. The 2nd generation cephalosporin which crosses the blood
brain barrier:

Page 152

Pharmacology 4th ed
Rang
Page 29

0.50

Pharmacology 4th ed
Rang
Page 308

0.25

Pharmacology 4th ed
Rang
Page 275

0.25

Pharmacology 4th ed
Rang
Page 290
Pharmacology 4th ed
Rang
Page 450

0.50

Pharmacology 4th ed

1.0

1.0

A. Cefoxime
C. Cefoxitin
B. Cefuroxime
D. Cefoperazone
25. A G2 P1 term, will be given prophylactic antibiotic prior to
CS. Which of the following should be recommended?
A. Cephalexin
C. Cefazolin
B. Cefoxitin
D. Ceftriaxone
26. After prolonged treatment with penicillin, the patient
developed pseudomembranous colitis. What antibiotic
will you give for this case?
A. Aztreonam
C. Imipenem
B. Ceftriazone
D. Vancomycin
27. This agent inhibits topoisomerase II (DNA gyrase)
A. Clindamycin
C. Spectinomycin
B. Ciprofloxacin
D. Tetracycline
28. A neonate was given IV antibiotics for 3 days. However,
the baby developed hypothermia, diarrhea and grayish
color of the skin. The baby was probably given:
A. Amikacin
C. Chloramphenicol
B. Erythromycin
D. Gentamycin
29. The purpose of giving compound drug therapy in the
treatment of tuberculosis:
A. to rapidly eradicate the strains of tubercle bacilli
B. to shorten the infectious phase
C. to decrease the emergence of resistant organisms
D. to prevent complications of tuberculosis
30. Which of the following is NOT a first line agent in the
treatment of tuberculosis?
A. Ethambutol
C. Rifampicin
B. Pyrazinamide
D. Streptomycin
32.A patient taking warfarin was also being treated for fungal
infection. After 7 days,the patient developed epistaxis. Which
of the following antifungal agents was given?
A.Nystatin
C.Flucytosine
B.Ketoconazole
D.Amphotericin B
33.If the physician makes a prescription order that utilizes a
drug supplied by the pharmaceutical company, what class of
prescription order was made?
A.Extemporaneous
C.Precompounded

Rang
Page 694
Pharmacology 4th ed
Rang
Page 695
Pharmacology 4th ed
Rang
Page 702

0.33

0.25

Pharmacology 4th ed
Rang
Page 720
Pharmacology 4th ed
Rang
Page 692

0.25

Pharmacology 4th ed
Rang
Page 706

0.25

Pharmacology 4th ed
Rang
Page 706
Pharmacology 4th ed
Rang
Page 317

1.0

Generics
Acts of the
Philippines

0.25

1.0

0.25

B.Compounded
D.Erroneous
34. Which among these antithrombotic drugs inhibit the
synthesis of thromboxane by irreversible acetylation of the
enzyme cyclooxygenase?
A. Abciximab
C. Integrilin
B. Aspirin
D. Ticlopidine
35. A 60 year old was seen at the OPD due to epistaxis which
occurred thrice for the past week and easy bruising. He is
taking warfarin for AF. Perinent Lab exam: INR-4.5
The INR result is brought about by which of the following?
A.increased thrombin activity
B.decreased platelet activity
C.diminished levels of vitamin K
D.decreased levels of factors II, VII,IX and X
36. A 67 year old with atrial fibrillation is currently taking
warfarin and digoxin. In the event this patient develops
massive bleeding, which of the following would be appropriate
for him?
A.give vitamin K
B.change to LMWH
C.give protamine sulfate
D.transfuse platelets
37.A 26 year old medical student was brought to the ER for
an apparent suicide poisoning due to a broken heart. The
maid could not recall the medication but claimed the bottle
contained medicine for headache. On PE, the physician noted
cyanosis of the oral mucosal, tongue and nail beds. Which of
the following is the drug most likely ingested?
A.Aspirin
C.Flurbiprofen
B.Acetaminophen
D.Meloxicam
38.A 52 year old woman was suffering from severe joint pains
and was diagnosed to have Rheumatoid Arthritis. Apparently,
she also appears to be suffering from acid peptic disease.
Among the following, which is the safest to give her?
A.Aspirin
C.Celecoxib
B.Ibuprofen
D.Phenylbutazone
39.A 21 year old male, took a bottle of insecticide from their
garden approximately 30-40 cc in amount. He later developed

Pharmacology
Goodman
and Gilman
10th ed.
Pp 1534
Pharmacology 4th ed
Rang
Page 316

0.50

Pharmacology 4th ed
Rang
Page 319

0.25

Pharmacology 4th ed
Rang
Page 232

0.25

Pharmacology 4th ed
Rang
Page 234

0.50

Pharmacology 4th ed

0.25

1.0

dizziness, visual blurring, generalized body weakness,


numbness, severe epigastric pain and shortness of breath. He
was found lying on the floor with stiffening of the trunk and
extremities and soaked with sweat. He was rushed to the ER,
pertinent findings included drooling of saliva with 3 episodes
of non projectile vomiting. Which of the following should be
given?
A.Pilocarpine
C.Atropine
D.Physostigmine
D.Carbachol
40.This inhibits de novo synthesis of cholesterol by inhibiting
HMG CoA reductaseA.Atorvastatin
C.Gemfribozil
B.Colestipol
D.Niacin
41.An asthmatic patient came to the ER because of
generalized wheezing in the lungs. Urgent management was
done. However, after a while, hypertension was noted. Which
of the following medications was probably given?
A.salbutamol
C.terbutaline
B.isoproterenol
D.salmeterol
42.A flight attendant has been having allergic rhinitis. She
comes to your clinic asking for medication prescription two
hours prior to her flight. What will be appropriate for her?
A.diphenhydramine
C.loratadine
B.chlorpheniramine
D.promethazine
43.This acts by inhibiting the immediate phase of asthma as it
prevents mediator release from mast cellsA.cromolyn sodium
C.montelukast
B.beclomethasone
D.zileuton
44.A patient with severe persistent asthma was treated for six
months now with prednisone. Which of the following would be
the expected side effect.
A.cushingoid facie
C.hyperkalemia
B.hypoglycemia
D.leonine facie
45.FM, 40 year old male was seen at the clinic for check-up.
He has bradykinesia, muscle rigidity, pill rolling movement,
resting tremors and shuffling gait. The single most effective
agent in the treatment of this disorder is:
A. Levodopa
C. Carbamazepine
B. Baclofen
D. Tacrine

Rang
Page 122

Pharmacology 4th ed
Rang
Page 305
Pharmacology 4th ed
Rang
Page 160

0.50

Pharmacology 4th ed
Rang
Page 345

0.50

Pharmacology 4th ed
Rang
Page 347
Pharmacology 4th ed
Rang
Page 423

0.25

Pharmacology 4th ed
Rang
Page 509

1.0

0.50

0.50

46. Which of the following is a selective inhibitor of MAO-B?


A. Selegeline
C. Tolcapone
B. Amantadine
D. Bromocriptine

Pharmacology 4th ed
Rang
Page 511
Pharmacology 4th ed
Rang
Page 510

0.50

47. Which of the following statement is NOT true regarding


Levodopa?
A. It is almost always used in combination with
decarboxylase inhibitors.
B. It has a short half-life about 1 3 hrs.
C. If administered with meals, its absorption is
delayed.
D. It has very little side effect and can be withdrawn
abruptly.
48. What effect does the use of oral contraceptive (OC) have
on a patient with hypothyroidism?
A. Low dose OC are unlikely to affect her thyroid
function tests.
B. Estrogen may falsely increase TSH due to alteration
in TSH secretion.
C. Estrogen may falsely decrease FT4 index due to
alterations in TBG
D. Estrogen may falsely elevate total T4 levels due to
alterations in TBG
49. If a hypothyroid patient pregnant, how might her
levothyroxine replacement be affected?
A. She might require an increase in dose.
B. She may need a decrease in dose.
C. She may need to stop the medication.
D. She may have to change levothyroxine to
liothyronine.
50. A 62 year old with atrial fibrillation was seen at the ER
with light headedness and palpitations associated with
exercise usually relieved by rest. At present his heart rate is
at 140/min. Which of the following is least likely effective in
controlling his heart rate?
A. Digoxin
C. Verapamil
B. Diltiazem
D. Atenolol
51. Patients receiving insulin therapy generally require which
of the following interventions with progression to end-stage
renal disease?

Pharmacology 4th ed
Rang
Page 630

0.25

Basic and
clinical
Pharmacol
ogy 9th
Edition
Katzung p
631
Pharmacology 4th ed
Rang
Page 265

0.25

Basic and
clinical
Pharmacol

0.25

0.25

0.25

A. decrease total insulin dose


B. increase total insulin dose
C. increase frequency of insulin administration
D. no change in insulin regimen
52. The inhibitor of GIT lipase is useful in the treatment of
obesity and has minimal side effects:
A. sibutramine
C. orlistat
B. phentermine
D. glucosan
53. An experiment on GI drugs was done. The baseline gastric
volume 2 cc and when tested with litmus paper, The paper
remained pink. After administering drug A, the gastric volume
was 2 cc and litmus paper turned blue. This means
A. drug A is effective in lowering gastric pH
B. drug A is effective in increasing gastric pH but no
effect on volume
C. The drug is effective in lowering the gastric pH and
volume
D. The drug has no effect on the volume and pH of
gastric juice
54.A patient taking this drug came into the clinic due to visual
disturbances, constipation, difficulty in urination and dry
mouth. This drug could be:
A.Muscarinic antagonist
B.histamine H2 antagonist
C.dopamine D2 antagonist
D.gastrin antagonist
55.The onset and duration of action of NPH insulin are
extended because:
A.protamine decreases the rate at which insulin is
absorbed
B.protamine blocks insulin metabolism in the liver
C.protamine is basic and combines with insulin by
charge interactions
D.protamine is slowly degraded proteolytically releasing
the bound insulin
56. An estrogen receptor antagonist which is useful for
postmenopausal breast cancer
A. Megastrol
C. Flutamide
B. Tamoxifen
D. Leuprolide

ogy 9th
Edition
Katzung p
694
Pharmacology 4th ed
Rang
Page 407
Pharmacology 4th ed
Rang
Page 375

0.25

0.25

Pharmacology 4th ed
Rang
Page 122

0.50

Basic and
Clinical
Pharmacol
ogy 9th
Ed.
Katzung
p.697

0.25

Basic and
Clinical
Pharmacol
ogy 9th
Ed.

0.50

57. The agent of choice in the treatment of gestational


diabetes
A. Sulfonylurea
C. Insulin
B. Biguanide
D. Acarbose
58. A 40 year old bank executive sought consult due to a
difficulty of going to sleep but no problem in maintaining
sleep. He stopped drinking coffee, eating chocolates, and
smoking a month ago. The most appropriate drug for him is:
A. Diphenhydramine
C. Phenobarbital
B. Midazolam
D. Clonazepam

59. A kidney transplant patient is given a drug for


immunosuppression to prevent rejection of the allografted
organ. The drug is most likely
A. Cyclosprine
C. Cyclophosphamide
B. Azathioprine
D. Vincristine
60. The preferred treatment for a patient who suffers a series
without complete recovery is:
A. Diazepam
C. Succinylcholine
B. Chlopromazine
D. Ethosuximide

61. A company manager is suffering from allergic rhinitis. He


has a series of staff meeting so he needs a medication that
will not make him drowsy. Which of the following would you
give?
A. Chlorpheniramine
C. Terfenadine
B. Hydroxyzine
D. Diphenhydramine

62. A patient with severe liver disease eventually developed


encephalopathy. Which agent is important in the therapy of
this condition?
A. Lactulose
C. Loperamide
B. Lactate
D. Lorazepam
63. A substance that enhances the probability of ovulation by
blocking the inhibitory effect of estrogen and thus stimulating

Katzung
p.679
Pharmacology 5th ed
Rang
Page 388
Basic and
Clinical
Pharmacol
ogy 9th
Ed.
Katzung
p.362
Pharmacology 5th ed
Rang
Page 257

1.0

0.33

0.33

Basic and
Clinical
Pharmacol
ogy 9th
Ed.
Katzung
p.391
Basic and
Clinical
Pharmacol
ogy 9th
Ed.
Katzung
p.266
Pharmacology 5th ed
Rang
Page 375

0.33

Basic and
Clinical

0.50

0.50

1.0

the release of gonadotrophin from the pituitary is:


A. Clomiphene citrate
C. Tamoxifen
B. Danazol
D. Progesterone
64. Which of the following combination (diuretic agent and
MOA) is CORRECT?
A. Furosemide inhibition of carbonic anhydrase
B. Acetazolamide osmotic effect
C. Hydrochlorothiazide inhibition of Na-Cl symport
D. Indapamide inhibition of Na-K-2Cl symport

65. Which of the following combination of diuretic and site of


action on the nephron is CORRECT?
A. Ethacrynic acid thick ascending limb of loop of
Henle
B. Furosemide proximal tubule
C. Hydrochlorothiazide late distal tubule
D. Spironolactone distal convoluted tubule
66. Which of the following is preferred for a pregnant woman
with hyperthyroidism?
A. Carbimazole
C. RAI
B. Methimazole
D. PTU

67. Drugs A and B act on the same tissue or organ through


independent receptors, but with opposite effects. This is
known as:
A. Competitive antagonism
B. Irreversible antagonism
C. Physiologic antagonism
D. Chemical antagonism
68. This pharmacokinetic value most reliably reflects the
amount of drug reaching the target tissue after oral
administration
A. Volume of distribution
B. Area under the blood concentration-time curve
C. Peak blood concentration
D. Time to peak blood concentration

Pharmacol
ogy 9th
Ed.
Katzung
p.681
Basic and
Clinical
Pharmacol
ogy 9th
Ed.
Katzung
p.249
Basic and
Clinical
Pharmacol
ogy 9th
Ed.
Katzung
p.242
Basic and
Clinical
Pharmacol
ogy 9th
Ed.
Katzung
p.632
Goodman
and Gilman
8th ed p
53-54

Goodman
and gilman
8th edition
Pp2627

0.50

0.50

0.33

0.5

0.5

69. A 3 year old patient passed out noodle-like worms and


was given an appropriate anti-helminthic. The
mechanism of the drug is most probably via
A. interference with cell wall synthesis
B. inhibition of neuromuscular transmission
C. interference with cell wall division
D. interference with protein synthesis
70. A patient with nosocomial pneumonia is given ceftazidime
and another antibiotic. The most probabale purpose for
adding the second antiobiotic is to:
A. Increase renal excretion
B. Decrease systemic toxicity
C. Increase oral absorption
D. Prevent the emergence of resistant bacteria
71. About 75% of patients in this condition may suffer from
vertigo, inability to perceive termination of movement and
difficulty in sitting or standing without visual clues
A. Patients allergic to penicillin
B. Patients receiving streptomycin therapy
C. Patients on isoniazid for tuberculosis
D. Patients on amphotericin B
72. A kidney transplant patient is given a drug for
immunosuppression to prevent rejection of the allografted
organ. The most appropriate drug is:
A. Cyclosporine
B. Azathioprine
C. Cyclophosphamide
D. Vincristine
73. A hypertensive patient is treated by a doctor and
experiences hemolytic anemia with a positive Coombs
test. This is most likely caused by:
A. Clonidine
C. Methyldopa
B. Captopril
D. Prazosin
74. In a hypertensive patient who is taking insulin to treat
diabetes, which of the following drugs should be used
with caution?
A. Propanolol
C. Hydralazine
B. Methyldopa
D. Prazosin
75. A patient has been taking Imipramine for depression.

Goodman
and gilman
8th edition
pp. 969970

0.5

Goodman
and gilman
8th edition
Pp1127

1.0

Goodman
and gilman
8th edition
Pp1104 1108

0.5

Goodman
and gilman
8th edition
Pp1270

0.5

DiPalma
4th ed p.
472

0.50

Goodman
and Gilman
8th ed
p.239

0.5

Goodman

1.0

He would most likely experience which of the following


adverse effects?
A. Seizures
C. hepatotxicity
B. anticholinergic effects
D. nephrotoxicity
76. Chlorpromazine may be used not only in treating
schizophrenia but is also effective
A. in reducing nausea and vomiting
B. as an antihistaminic
C. as an antihypertensive agent
D. for treating bipolar affective disorder
77. A child suffers from lapses of consciousness or vacant
stares lasting about 10 secs, often in clusters. The
drug which may be most helpful for his disorder is:
A. Phenytoin
C. Ethosuximide
B. Carbamazepine
D. Phenobarbital
78. Upon taking Penicillin, a patient suddenly experiences
respiratory distress and circulatory collapse. Which
drug would be most helpful in this situation?
A. Norepinephrine
C. Isoproterenol
B. Epinephrine
D. Atropine
79. A gout patient developed nephropathy characterized
by overproduction of uric acid and extreme
hyperuricemia. What agent could have helped prevent
this?
A. Cyclophosphamide
C. Sodium chloride
B. Allopurinol
D. Antidiuretic hormone
80. A patient with peptic ulcer and on therapy with
cimetidine may experience decreased effects of many
other drugs because cimetidine is an inhibitor of
A. the proton pump
C. monoamine oxidase
B. tyrosine kinase
D. cytochrome P-450
73. The index of safety of a drug decreases as the median
toxic dose:
A. decreases and the median effective dose increases
B. increases and the median effective dose decreases
C. the median effective dose decreases
D. the median effective dose increases
74. What is the major second messenger of beta receptor

and Gilman
8th ed
p.405-414
A

activation that participates in signal transduction?


A. inositol triphosphates
C. calcium
B. cAMP
D. adenylyl cyclase

DiPalma
4th ed
p.275-277

0.5

Goodman
and Gilman
8th p. 449
453

0.5

Goodman
and Gilman
8th ed
p.198

1.0

Goodman
and Gilman
8th ed
p.678

1.0

Goodman
and Gilman
8th ed
p.901

1.0

Basic and
Clinical
Pharmacol
ogy 9th
Ed.
Katzung
p.30
Basic and

0.33

0.5

75. Which of the following bronchodilators is a xanthine


derivative?
A. Terbutaline
C. theophylline
B. ipratropium bromide
D. salmeterol

76. Myelosuppression is a common side effect associated


with the use of the following anticancer drugs,
EXCEPT:
A. Vincristine
C. Methotrexate
B. Cyclophosphamide
D. Chlorambucil

77. The following anticancer drug, with its toxic effect, is


correctly matched with the appropriate antidote:
A. cyclophosphamide-induced hemorrhagic cystitis
ondansetron
B. methotrexate-induced myelosuppression MESNA
C. cisplatin-induced nausea and vomiting leucovorin
D. doxorubicin-induced arrhythmia - dexrazoxane
78. Which of the following anticancer drugs act as a
mitotic inhibitor?
A. Methotrexate
C. Bleomycin
B. Vinblastine
D. Carmustine

79. Radical cure for malaria is achieved by giving:


A. Chloroquine
C. Mefloquine
B. Quinine
D. Primaquine

Clinical
Pharmacol
ogy 9th
Ed.
Katzung
p.25-27
Basic and
Clinical
Pharmacol
ogy 9th
Ed.
Katzung
p.324,
325, 328
Basic and
Clinical
Pharmacol
ogy 9th
Ed.
Katzung
p.911
Basic and
Clinical
Pharmacol
ogy 9th
Ed.
Katzung
p.914
Basic and
Clinical
Pharmacol
ogy 9th
Ed.
Katzung
p.911
Basic and
Clinical
Pharmacol
ogy 9th
Ed.

1.0

0.33

0.50

0.75

1.0

80. A 3 year old was brought to the ER because of


recurrent attacks of blank stares lasting for a few
minutes. The anti-seizure agent that you will give the
child is:
A. Phenobarbital
C. Lamotrigine
B. Ethosuximide
D. Vigabantrine
81. The mainstay drug in the treatment of all types of
leprosy is:
A. Clofazime
C. Isoniazid
B. Dapsone
D. Rifampicin
82. A patient on anti-TB drug developed peripheral
neuritis. This was improved by giving Pyridoxine. The
drug taken must have been:
A. Ethambutol
C. Isoniazid
B. Rifampicin
D. Para-aminosalicylic acid

73. A patient with elevated cholesterol level is also taking


warfarin for his deep venous thrombosis. The lipid
lowering agent that may enhance the effect of
Warfarin is:
A. nicotinic acid
C. cholestyramine
B. lovastatin
D. gemfibrozil

74. A known diabetic patient was brought to the ER


unconscious with random blood sugar of 900 mg/dl.
ABG study showed metabolic acidosis. Which of the
following preparation is appropriate?
A. regular insulin
B. insulin zinc preparation
C. isophane zinc suspension
D. potassium zinc insulin suspension
75. Which of the following corticosteroids has the greatest
anti-inflammatory potency relative to cortisol?
A. Prednisone
C. Aldosterone
B. Dexamehtasone
D. Triamcinolone

Katzung
p.871
Goodman
and Gilman
10th ed
p.535

0.25

Goodman
and Gilman
10th ed
p.1288
Basic and
Clinical
Pharmacol
ogy 9th
Ed.
Katzung
p.784
Basic and
Clinical
Pharmacol
ogy 9th
Ed.
Katzung
p.574
Basic and
Clinical
Pharmacol
ogy 9th
Ed.
Katzung
p.700

1.0

Basic and
Clinical
Pharmacol
ogy 9th
Ed.

0.5

1.0

0.5

0.75

76. A 30-year old G2P1 patient suffers experiences a


regular and increasingly frequent contractions at 6
months of gestation. Which of the following agents
may help in this case?
A. Terbutaline
C. Propanolol
B. Nicotine
D. Tacrine

77. The primary mechanism underlying the resistance of


gram positive organism to macrolide antibiotic is:
A. methylation of binding site on the 50S ribosomal
subunits
B. formation of esterases that hydrolyzed the lactone
ring
C. formation of drug-inactivating acetyltransferase
D. decrease drug permeability of the cytoplasmic
membrane
78. A prescription that has the generic name enclosed in a
parenthesis and written below the brand name can be
interpreted as:
A. violative prescription
B. imposiible prescription
C. erroneous prescription
D. correct prescription
79. The onset and duration of action NPH insulin are
extended by protamine because:
A. it decreases the rate of insulin absorption
B. it blocks insulin metabolism in the liver
C. it is a basic compound and combines with insulin
by charge interactions
D. it is slowly degraded proteolytically releasing the
bound insulin
80. The following anti-anginal drugs are vasodilators,
EXCEPT:
A. Felodipine
C. Metoprolol
B. Nitroglycerin
D. Diltiazem

Katzung
p.647
Basic and
Clinical
Pharmacol
ogy 9th
Ed.
Katzung
p.138
Goodman
and Gilman
10th ed
p.1250

1.0

0.25

Generic
Act of the
Philippines

0.5

Pharmacology 4th ed
Rang
Page 697

0.25

Basic and
Clinical
Pharmacol
ogy 9th
Ed.
Katzung
p.186

0.50

81. Which is a direct effect of nitrates when given in the


usual doses for the management of angina?
A. increased preload
B. increased afterload
C. increased cardiac contractility
D. decreased preload

73. The mechanism of action of Omeprazole


A. inhibits H+-K+ ATPase pump
B. promotes prostaglandin formation
C. blocks action of histamine of H2 receptors
D. neutralizes acid by increasing the pH of gastric
secretion

Basic and
Clinical
Pharmacol
ogy 9th
Ed.
Katzung
p.190
Goodman
and Gilman
8th ed
p.902-904

1. 3rd generation cephalosporin that does not cross the BBB


a. Cefoperazone (and Cefipime)
b. Ceftazidime
c. Moxolocam
d. cefetamet
e. all of the above
2. associated with pseudomembronous colitis;
a. methicillin
b. Pen G Na
c. Oxacillin
d. Ampicillin
e. Ticarcillin
3. Tetracyclines are contraindicated in children because it may causes;
a. GI irritation
b. Hepatotoxicity
c. Bone and tooth discoloration
d. Renal toxicity
e. Only A and B
4. Anticonvulsants that have been shown to have antimanic effects;
a. Carbamazepine
b. Valproic acid
c. Phenytoin

0.50

1.0

d. All of the above


e. A and B only
5. The major working hypothesis for schizophrenia, and the basis for much
of drug therapy is the;
a. Serotonin hypothesis
b. Omine hypothesis
c. Dopamine hypothesis
d. Muscarinic hypothesis
e. Serotonim hypothesis
6. Causes harmless orange color of urine ,sweat and saliva;
a. INH
b. Rifampin
c. Ethambutol
d. Cycloserine
e. Ethionamide
7. This anticancer causes a minimal bone suppressant effect;
a. Vincristine
b. Methotrexate
c. Cyclophosphamide
d. Cytarabine
e. Doxorubucin
8. This is the drug of choice for mixed round worm infection;
a. Mebendazole
b. Pyrantel pamoate
c. Niclosomide
d. Thiobendazole
e. Metronidazole
9. The fraction of the drug that reaches the general circulation is;
a. Bioavalability
b. Bioequivalence
c. Biodisposition
d. Biotransformation
e. First pass
10. This deals with the biochemical and physiological effects of drugs and
their mechanism of action
a. Pharmacokinetics
b. Pharmacodynamics

c. Pharmacotherapeutics
d. Pharmacogenetics
e. Pharmacy
11. Which one of the following pairs of drugs has been known to cause a
lupus like syndrome
a. Guarenthidine and reserpine
b. Lidocaine and phenytoin
c. Procainamide and hydralazine
d. Proprendol and disopyramide
12. Which one of the following statements is false about furosemide? It
a. Increases the excretion of Na, Cl, and water
b. Increases the excretion of K
c. Increases the excretion of uric acid
d. May have to be used in conjunction with K sparing diuretic
13. The treatment of ketoacidosis may include the administration of
a. CaCl
b. Crystalline zinc insulin
c. Glucagons
d. ZnCl
14. The action of cortisone is characterized by
a. Causing hyperglycemia
b. Decreasing gluconeogenesis
c. Enchancing peripheral utilization of glucose
d. Having an antiglucagon effect
15. Methimazole exerts its effects by inhibiting;
a. Iodide acidification
b. Iodide transport and accumulation
c. The formation of diiodothyrosine
d. The tubular secretion of iodine
16.

16.

Pharmacological properties of 5HT except:

a. Enhances GIT motility


b. Increased force of contraction in the heart
c. Bronchoconstriction
d. Bronchodilation
Piperazine derivatives except
a. Hydroxyzine
b. Cetirizine

c. Meclizine
d. Carbocisteine (an opioid)
17.
They do not undergo the process of phosphorylation to exert their
anti viral activity
a. Indinavir (protease inhibitor)
b. Cidofavir
c. Epivir
d. Lamivudine
18.
The following drugs reduce platelet aggregation by inhibiting the ADP
pathway
a. Clopidogrel (& Ticlopidine)
b. Abciximab
c. Aspirin
19.
Which drug decreases plasma triglycerides by increasing lipoprotein
lipase;
a. Lovastatin
b. Nicotinic acid
c. Cholestyramine
d. Probocol
e. clofibrate
20.
Inhibits phosphodiesterase enzyme
a. Salbutamol
b. Aminophylline
c. Cromolyn sodium
d. Ephedrine
e. B & C
21.
Agent of choice in congestive heart failure which are superior to other
vasodilators
a. Felodipine
b. Spironolactone
c. Enalapril
d. Digoxin
22.
Occurs during phase 2 of action potential of Purkinje fibers
a. Upstroke ends as Na channels are rapidly inactivated
b. K channels rapidly open and close causing a transient
outward current

c. Voltage-sensitive Ca-channels upon resulting in slow


inward current
d. K-channels open resulting outward current
23.
Orally given in patients with severe left ventricular systolic
dysfunction
a. Digoxin
b. Dobutamine
c. Oraninone
d. A and B only
24.
An agent that neutralizes HCI secreted by parietal cells and
increases gastric motility
a. Aluminum hydroxide
b. Magnesium hydroxide
c. Calcium carbonate
d. Sodium bicarbonate
e. Calcium chloride
25.
Contraindications for aspirin use except;
a. Elderly
b. PUD
c. Pregnancy
d. Hemophilia
e. Liver damage
26.
A drug used in the treatment of gout that reduces uric acid
synthesis by blockage of hypoxanthine;
a. Indomethacin
b. Aspirin
c. Colchicines
d. Allopurinol
e. Sulfinpyrazone
27.
The following is are True about Naftifine;
a. Interferes with ergosterol biosynthesis
b. Inhibits squalene epoxidase
c. Effects CYP 450 enzyme system
d. Significant drug interact
e. A and B only
28.
Inhibit DNA polymerase enzymes
a. Didanosine

b. Stavudine
c. vidarabine
d. AZT
e. A and C only
29.
Specific 5HT3 antagonist used for nausea and vomiting secondary
to chemo therapy
a. Methylgerside
b. Ondansetron
c. Metoclopromide
30.
H1-receptor antagonist is which has anti-cholinergic properties
except
a. Diphenhydramine
b. Promethazine
c. Meclizine
d. Estemizole
31.
A patient was diagnosed as suffering from chronic refractory
congestion and oliguria. Which one of the following agents is most
effective?
a. Acetylcholine
b. Atropine
c. Dopamine
d. Terbutaline
e. Propanolol
32.
The nonselective beta-adrenergic blocking agent that is also a
competitive alpha-adrenoceptors
a. Timolol
b. Madolol
c. Pindolol
d. Labetolol
e. Esmolol
33.
All of the following compounds are direct-acting cholinergic
agonist, except
a. Acetylcholine
b. Pilocorpine
c. Edrophonium
d. Bethanecol
34.
Atropine and its analogs produce which of the following effects;

a. Pupillary constriction
b. Increased gastric acid secretion
c. Decreased secretion in the respiratory tract
d. Increased peristalsis
e. Bronchoconstriction
The most potent of the inhalational anesthetics is;
a. Enflurane
b. Halothane
c. Isoflurane
d. Methoxyflurane
An ester local anesthetic with vasoconstrictive properties:
a. Lidocaine
b. Tetrocaine
c. Bupivacaine
d. Cocaine
e. Procaine
Drug of choice for management of acid hypersecretion associated

35.

36.

37.
with
38.

39.

40.

41.

a. Cholinergic agonist
b. Prostaglandin analog
An antiarrhythmic drug that acts on K-channels
a. Esmolol
b. Sotolol
c. Diltiazem
d. Adenosine
Prolongs phase 3 repolorization
a. Na-channel blocker
b. B-adrenareceptor blocker
c. K-channel blocker
d. Ca-channel blocker
True statements about lidocaine except
a. Use for ventricular arrhythmias arising from MI
b. Does not markedly slow conduction
c. More effective on atrial or AV junction
arrhythmias
d. None of the above
Calcium channel blocker which can relieve coronary spasm

a. Mifedipine
b. Verapomil
c. Diltiazem
d. Hydralazine
42.
Commonly used antihypertensive drugs in patients with insulin
dependent diabetes mellitus
a. Calcium channel blockers
b. ACE inhibitors
c. Both of the above
43.
Effects of angiotensin II on adrenal cortex
a. Stimulates catecholamine biosynthesis
b. Stimulates mineralocorticoid synthesis
c. Renal vasoconstriction
44. Specific for angiotensin AT receptors
a. Captopril
b. Candesertan
c. Voscose
d. A and B only
45.
Overuse of loop diuretics will cause which of the following
a. Hypokalemic metabolic alkalosis
b. Hyperkalemic metabolic acidosis
c. Hypokalemic metabolic acidosis
d. Hyperkalemic metabolic alkalosis
46. The above condition is due to the secretion of
a. Sodium and hydrogen ions
b. Potassium and hydrogen ions
c. Sodium and chloride ions
d. Potassium and chloride ions
47. Which of the following should not be given to a hypertensive
a. Lisinopril
b. verapamil
c. methyldopa
d. Hydrochlorothiazide
48.Inhibits platelet aggregation by inhibiting ADP pathway
a. Aspirin
b. Heparin
c. Warfarin
d. Ticlopidine

49.

Inhibits ligand binding to platelet GP IIb/IIIa receptor by occupancy


a. Abxicimab
b. Tirofiban
c. Enoxaparin
d. Anistreplase
50. Analog of aminocaprioc acid that competitively inhibits plasminogen
activated
a. Tranexamic acid
b. Eprotinim
c. Reteplase
d. Warfarin
51. Effective as a singe drug therapy for familial dysbetalipoproteinemia
a. Fibric acid
b. Bile acid-binding resin
c. HMG-CoA reductase
d. Probucol
52. Most common side effect of antihyperlipidemic agents
a. Elevated blood pressure
b. Neurological problems
c. Gastrointestinal disturbance
d. Migraine headaches
53. Causes decrease in liver triacylglycerol synthesis by limiting available
free fatty acid needed as building blocks for this pathway
a. Cholestyramine
b. Elefibrate
c. Levostatin
d. Niacin
54. The following drugs maybe used during acute exacerbations of bronchial
asthma except
a. Terbutaline
b. Salmeterol
c. Albuterol
d. Epinephrine
55. Effective as prophylactic anti-inflamatory agent
a. Nedocromil
b. Ipratropium
c. Theophylline

d. Loratidine
56. Most reabsorption occurs at what segment of the nephron
a. Distal convoluted tubule
b. Loop of Henle
c. Collecting duct
d. Proximal convoluted tubule
57. Are sulfonamide derived diuretic which inhibits Na+/CI- cotransporter at
the DCT
a. Ethocrymic acid
b. Acetazolormide
c. Chlorthiazide
d. Triamtereme
58. Gynecomastia is an adverse effect of this diuretic
a. Mannitol
b. Spironolactone
c. Chlorthalidone
d. Bumetinide
59. H2 receptor antagonist eliminated principally by the kidney hence has
100% bioavailability
a. Cimetidine
b. Nizatidine
c. Famotidine
d. Ranitidine
60. A target plasma theophylline concentration of 5mg/l is desired to
relieve acute exacerbation of bronchial asthma in a 70 kg patient. If the
mean clearance of this drug is 2800 ml/kg, what is the dose rate to be
given via IV?
a. 7 mg/h
b. 12mg/h
c. 28 mg/h
d. 14mg/h (2.8L/kg X 70kg)
61. Exacerbation was relieved and you want to maintain the plasma level
rising oral theophyline every 12 hours using an extended release
formula. If oral availlability is 96% what is the maintenance dose?
a. 175 mg/dose
b. 350 mg/dose
c. 700 mg/dose

d. 700 mg/day
62. Single most important factor determining drug concentration
a. Clearance
b. Half-life
c. Absorption
d. Volume of distribution
63. A 15 year old maintained on phenobarbital was diagnosed to have
typhoid fever. Which of the following antibiotic dose should be increased
to achieve a therapeutic dose?
a. Trimethoprim
b. Sulfamethoxazole
c. Chloramphenicol
d. Ofloxacin
e. Paracetamol
64. MG. 38 year old female, 180 lbs, wanted to reduce weight by taking
grapefruit juice. Which of the following drug metabolism will be
inhibited?
a. Astorvastatin
b. Cisapride
c. Dicumerol
d. A and B only
e. All of the above
65. Active component of the above substance inactivates
a. Intestinal CYP3A4
b. Hepatic CYP3A4
c. Both
d. Neither
66. This/these refer/s to the concentration or those of drug required to
produce 50 maximal effect
a. Potency
b. Efficacy
c. Graded dose response
d. Quintal dose effect
67.Response to a drug diminishes rapidly after administration of a drug
a. Tolerance
b. Tachyphylaxis
c. Idiosyncracy

d. Median effect
A synthetic derivative of glycyrrhizic acid that shows effective healing
of gastric duodenal ulcers:
a. Sucrolfate
b. Carbenaxolone
c. Octreotide
d. Pirenzipine
69. Emulsifies with stool making passage easier
a. Docusate
b. Lactulose
c. Bisacodyl
d. Phenolphthalein
70.
Zileuton inhibits lipoxygenase pathway by
a. Inhibiting 5-lipoxygenase
b. Antagonizing LTD4-receptor
c. Alpha adrenoreceptor blockade
d. Inhibiting phospholipase A2
71.
Glucose transporter seen in muscle and adipose tissue which is
responsible for intake uptake of glucose.
a. GLUT 4
b. GLUT 3
c. GLUT 2
d. GLUT 1
72. Long acting insulin/s
a. Lente humulin
b. Insulin lispro
c. Ultralente humulin U
73. Beta-lactam antibiotic of choice for enterobacter infections:
a. Tazobactom
b. Meropenem
c. Aztreonom
d. Vancomycim
74.Tetracyclines maybe used to the following organisms except
a. Mycoplasma pneumoniae
b. Entameoeba histolytica
c. Neisseria gonorrhea
d. Helicobacter pylori
68.

75. Inhibits translocation by irreversibly binding to 50S ribosomal subunit


a. Azithromycin
b. Gentamycin
c. Fluroquinolone
76.A tissue Schizonticide
a. Chloroquine
b. Quinine
c. Mefloquine
d. Primaquine
77.Effective single agent for suppressing and curing multidrug resistant P.
falciparum
a. Quinine
b. Mefloquine (???NOTA!)
c. Pyrimethamine
d. Primaquine
78.Spectrum of activity of ribavirin
a. Rhinoviruses
b. Enteroviruses
c. Influenza viruses
d. Hepatitis
79.Dideoxynucleoside that terminates the synthesis of proviral DNA chain
and inhibits the RNA transcriptase of HIV and Hepatitis B virus
a. Stavudine
b. Lamivudine
c. Zalcitabine
d. Saquinavir
80.Alkylating agents used as anticancer drugs except
a. Paclitaxel (Mitosis inhibitor)
b. Cyclophosphamide
c. Nitrosureas
d. Mechlorethamine
81.A co-transmitter at inhibitory enteric nervous system neuromuscular
junction that is important at sphincters:
a. GABA
b. Golanin
c. 5 HT
d. NO

e. CGRP
82. An autonomic receptor which results to opening of potassium channels
and inhibits adenyl cyclase when activated
a. MI
b. M2
c. M3
d. NN
e. NM
83. An alpha adrenergic receptor blocker
a. Phentolamine
b. Isoproterenol
c. Norepinephrine
d. Betanechol
84. A direct acting cholinomimetic drug which is/are alkaloid or synthetic
analog
a. Carbonic acid
b. Pilocarpine
c. Carbachol
d. All of the above
85. The drugs above causes
a. Cillary muscle relaxation
b. Negative chronotrophy
c. Positive chronotrophy
d. Vasoconstriction at low dose
86. After a vehicular accident , the patient developed neurogenic bladder.
A cholinomimetic commonly used in this case is
a. Betanechol
b. Nicotine
c. Muscarine
d. Neostigmine
87. Before giving the drug above you must make sure that
a. There is no mechanical obstruction
b. The level of cord injury is exact
c. The patient had urinary retention
d. B and C
e. All of the above

88. A new direct acting muscarinic agonist used in the treatment of dry
mouth associated with Sjogren syndrome
a. Embenorium
b. Denepezil
c. Cevimeline (?di ko rin mabasa)
d. Tacrine
e. Pyridostigmine
89. A semisynthetic derivative of plant alkaloid podophyllatoxin blocking
cells in the late S-G1 of cell cycle
a. Etoposide
b. Procarbazine
c. L-asparaginase
d. Cisplatin
90. True of aspirin
a. Reversible acetylators of cyclooxgenase
b. Depresses pain stimull at subcortical sites
c. Antinflammatory, antipyretic, antiplatelet, analgesic
d. Responsible for keeping PDA open
91. An imidazopyridine derivative with hypnotic effects and facilitates GABA
mediated inhibition
a. Zolpidem
b. Buspirone
c. Zaleplon
d. Flumazeoil
92. Which of the ff. drugs is generally considered the drug of choice in
treating status epilepticus
a. Phenoborbital
b. Amoborbital (Amstal)
c. Phenytoin (dilantin)
d. Paraldehyde
e. Diazepam (Valium)
93. The antihypertensive effect of guanethidine (Ismelin) is inhibited by
a. Diazepam (Valium)
b. Amitriptyline
c. Hydrochlorothiazide (Hydro DIURIL)
d. Probenecid (Benemid)
e. Nitrofurantoin (furodentin)

94. A physician has decided upon a course of tetracycline therapy for a


patient with renal impairment. Which of the following drugs is LEAST
likely to accumulate in the blood
a. Demeclocycline
b. Doxycycline
c. Minocycline
d. Oxytetracycline
95.Benztropine (Cogentin) is often given to patients taking the antipsychotic
phenothiazine. Benztropine:
a. Reduces the doze of phenothiazine required
b. Is an anticholinergic drug that reduces the extrapyramidal
side effects of phenothiazine
c. Eliminates the unpleasant GI irritation caused by the
phenothiazine
d. Is an antidepressant
e.reduces gut motility to ensure that the phenothiazine is
completely absorb
92. The metabolism of which of the following compounds is altered in patients taking antimetabolite

a.
b.
c.
d.
e.

Pyridoxine
Folic acid
Riboflavin
Rennin
Tyroxine

93. Which of the following antifungal agents is ineffective against candida organism?
a. nystatin (mycostatin)
d. halopragin (halotex)
b. clotrimazole (lotrimin)
e. miconazole
c. tolnaftate (tinactin)
94. The clinical investigation of a new drug consists of four phases. Phase I of the clinical testing involves
administering the drug.
a. to animals to determine side effects of the drug
b. to animals for toxicity studies
c. by select clinicians to healthy volunteers
d. by select clinicians to patients suffering from the disease
e. by general practitioners to patients suffering from the disease
95. Tubocurarine should NOT be used in patients who are taking
a. aspirin

b. morphine
c. indomethacin
d. levodopa
e. gentamicin
96. Penicilloyl-polytysine is a substance used to:
a. stabilized crystalline penicililin G preparations
b. counteract allergic reaction to penicillin
c. reduce the renal secretion of penicillin
d. skin test patients for penicillin allergy
e. manufacture the semisynthethic penicillins
97. Ticarcillin may be preferred to carbenenicillin for patients with congestive heart failure, renal failure, and
hypertension because it
a. contains less sodium than does corbenicillin
b. is usually given in smaller doses than carbenicillin
c. does not induce hypertension
d. stimulates renal blood flow
e. has a broader spectrum than does carbenicillin
98. A Fanconi-like syndrome has been associated with the use of outdated and degraded
a. ampicillin
b. cloxycycline
c. tetracycline
d. chloromphenicol
e. clindemycline
99. Which of the following is true of lithium carbonate (Eskolith, Lithane)
a. Indicated in the treatment of severe manic-depressive psychoses
b. May only be administered by the intremuscular route
c. Onset of action occurs within 2 hours of the first administered dose
d. Should be administered with a diuretic to minimize edema formation
100. Patients on lithium carbonate therapy should be advised
a. to limit water intake
b. to stop taking the drug if they experience mild side effects
c. not to restrict their normal dietary salt intake
d. not to take the drug during the manic phase of their cycle
e. not to take the drug with food
101. Which of the following drugs can interfere with the diagnosis of pernicious anemia
a. Pyridoxine
b. Menadione
c. Thiamine
d. Ascorbic Acid
e. Folic Acid
102. Which of the following agents would be most dangerous to use in a patient with high doses of
gentamicin?

a. ethacrymic acid (Edecrin)


b. tetracycline HCI
c. propantheline bromide (pro-banthine)
d. hydroOIURIL
e. pentobarbital sodium
103. Diethylstilbestrol induced vaginal adenocarcinoma is an example of
a. Delayed irreversible toxicity
b. dose dependent toxicity
c. hypersensitivity reaction
d. idiosyneratic reaction
104. Ethylene glycol poisoning would lastly manifest as
a. acidosis
b. alkalosis
c. oxalate crystals in the urine
d.renal insufficiency
105. Isopropyl alcohol poisoning resembles the poisoning produced by which one of
a. ethyl alcohol
b. ethylene glycol
c. kerosene
d. methyl alcohol
102. Benzene toxicity is characterized by
a. anemia
b. otoxia
c. gastrointestinal symptoms
d. respiratory alkolocis
103. Immunnosuppressive agents such as prednisone and azothioprine have been..for all of the following
disorders, except which one?
a. active hepatitis
b. megalobiastic anemia
c. thrombocytopenic purpura
104. The metabolite of azathioprine is
a. Cloflucarbon
b. 6-mercaptopurine
c. merodiceire

d. triclosamide
105. The chief side effect of azathioprine is

d. sulfoxone
111. Primaquine is characterized by the following characteristic, except which one ?

a. alopecia

a. attacks plasmodia in exoerythrocytic stages

b. leucopenia

b. is effective for malarial prophylaxis

c. hypertrichosis

c. may cause hemolytic anemia in susceptible patients

106. The toxicity of trimethadione is manifested as which of the following


a. alopecia

d. may cause polycythemia vera


112. Thiabendozole is characterized by the which of the following

b. bone narrow depression

a. exerts its effects by inhibiting fumarate reductase

c. hemerolopia

b. is metabolized by dehydration

107. One reason nitrosoureas are effective in the treatment of brain tumors

113. Third generation cephalosporins such as cefotaxime and ceftazidime

a. are lipid soluble

a. are effective in treating infection caused by aerobic gram-negative

b. chelate carbonium ions

b. do not require dose adjustment in patients suffering from renal imparement

c. chelate isocyanate molecules

c. have identical pharmacokinetic properties

d. inhibit the generation of carbonium ions

d. require dose adjustment in patients with hepatic diseases

108. 5-Fluorouracil is given intravenously because it is


a. a gastric secretogogue
b. metabolized rapidly in the liver
c. not absorbed orally
d. a potent irritant to the gastric mucous
109. In brain tumors, which one of the following cells is deficient?
a. astrocytes
b. eosinophils
c. monocytes
d. T4- helper lymphocytes
110. Leprosy is treated mainly with
a. guenobenz
b. Guanfocine
c. methylphenidate

114. All of the ff. chemotherapeutic agents have been properly matched with their characteristics except one,
which one?
a. azlocillin is less potent than carbenicillin
b. carbenicillin indanyl- is effective orally
c. mezlocillin is effective in Klebsiella infection
d. ticarcillin is more potent than carbenicillin
115. Which one of the following penicillin derivatives is not stable in gastric acid
a. amoxicillin
b. dicloxicillin
c. penicillin G
116. Chloramphenicol is characterized by all of the following actions, except which one?
a. bacterial resistance results from the development of chloramphenicol acetyltransferases
b. it has a narrow spectrum of activity
c . it is a drug of choise in the treatment of salmonella infection

d. it is used for treating meningococoal onfection in patients allergic to penicillin


118. A 30 year old overweight woman suffered from paroxysmal supraventricular tachycardia and also
borderline hypertension (145/90mml-ig). For which she was not taking any medications. The cardiologist
should treat the tachycardia with which one of the following
a. digitals followed by quinidine
b. propanolol
c. propanolol plus thiazide
d. verapamil
118. All of the following agents participate in secretion in the exocrine pancreas, except
a. acetylcholine
b. cholecystokinin
c. oxytocin
d. substance P
119. All of the following agents belong to the phenylalkylamine derived calcium entry family except one,
which one
a. gallopomil
b. oxodipine

122. The pertussis toxin


a. alters the activity of adenylate cyclase
b. resembles botulinum toxin in action
c. resembles the cholera toxin in action
d. resembles forskolin in action
123. Activation of all of the ff. receptor sites causes stimulation of adenylate cyclase except one, which one?
a. alpha2- adrenergic receptors
b. beta- adrenenergic receptors
c. D1 dopaminergic receptors
d. serotonerg ic receptors
124. Which one of the ff. best describes the actions of morphine and enkephalin on adenylate cyclase
a. activate adenylate cyclase in therapeutic doses
b. inhibit adenylate cyclase in toxic doses
c. produce a selective and naloxone-reversible inhibition of adenylate cyclase
d. produce a nonspecific and naloxone-insensitive activation of adenylate cyclase
125. Which one of the ff. substances does not increase the concentration of cyclic AMP

c. ronipamil

a. aminophylline

e. verapamil

b. epinephrine

120. All of the ff. agents are dihydropyridine derived calcium entry blockers:
a. anipamil
b. nicardipine

c. glucagon
d. insulin
126. Renal cell carcinoma-induced hypercalcemia may be reduced by the administration of the ff. agents?

c. nifedipine

a. aspirin

d. riodine

b. calcitonin

121. Which one of the ff. is the least recommended indication for verapamil
a. atrial flutter
b. atrial fibrillation

c. parathyroid hormone
d. sodium citrate
126. The prostacyclin-induced secretion of renin is shared by which one of the ff.

c. digitalis toxicity

a. acetylsalicylic acid

d. reentrant paroxysmat supreventricular

b. indomethacin

c. meclofenomate
e. norepinephrine
127. The bronchodilating action of prostaglandin E1 is shared by
a. acetylcholine
b. bradykinin
c. histamine
d. protoglandin E2
128. In a patient recovering from an acute myocardial infection who is taking warfarin the administration of
clofibrate may

131. Following the administration of nitroglycerin, the change in cardiac hemodymanic is due to
a. constriction of collateral vessels
b. decreasing of myocardial oxygen consumption
c. shifting of the blood away from ischemic areas
d. venous dilation then increased venous return then rapid ventricular volume with increased wall
tension;
132. Antitussive preparations should be used
a. in productive cough
b. in unproductive cough

a. decrease the risk of hemorrhage

c. only when hypoxia is imminent

b. have no effect at all

d. in copious secrotory cough

c. increase the risk of hemorrhage


d. lower the vitamin K level
128. Alpha-tocopherol

133. All of the following drugs have antitussive properties except which one
a. codeine
b. deserpidine

a. causes thromboembolic diseases

c. dextromethorphan

b. inhibits platelet aggregation

d. morphine

c. is a natural oxidant (hindi daw natural)


d. is similar to alpha-interferon
129. Streptokinase and urokinase are contraindicated in the presence of

134. Which one of the following phenothiazine derivatives has no antiemetic effect?
a. chlorpromazine
b. prochlorperazine

a. arteriovenous shunt

c. thioridazine

b. malignancy

d. triflupromazine

c. pulmonary embolism
d. thrombosis
130. Transderm- Nitro is contraindicated in:

135. Which one of the ff. gastrointestinal agents does not stimulate parietal cell function
a. acetylcholine
b. gastrin

a. effort angina

c. histamine

b. elevated ocular and cerebrospinal fluid pressure

d. prostaglandin

c. stable angina
d. variant angina

136. Which one of the following compounds has both estrogenic and antrestrogenic properties
a. chlorpromazine

b. clofibrate

b. doxorubicin

c. clomiphene

c. dactinomycin

d. clonidine

d. cyclosporine

137. Which one of the ff/ agents inhibits spermatogenesis?

142. Urine monoclonal antibody to CD3 antigen on the surface of human thymocytes

a. gelusil

a. levarmisole

b. gemcodial

b. hymphocyte immune globulin

c. gestodene

d. muromonab

d. gossypol
138. The cellular metabolism of calcium is characterized by

143. Elective MAO type B inhibitor used as adjunct to parkinsonism


a. amantadine

a. calcium pumps maintaining the differential intracellular versus extracellular calcium

b. selegiline

b. being unaffected by anticonvulsants

c. bromocriptine

c. the intracellular calcium being bound only to organic phosphate

d. haloperidol

d. the intracellular concentration of calcium being 1,000-fold higher than the extracellular concentration
of calcium
139. Strategy in cancer chemotherapy wherin cancer cells are held at M phase followed by administration
with another CCS drugs
a. pulse therapy
b. recruitment
c. synchrony
d. rescue therapy
140. Cell cycle specific drug that acts in late S and Go phases which are used as a component of combination
therapy for testicular carcinoma
a. etoposide
b. flutamide
c. leuprolide
d. mercoptopurine
141. Peptide antibiotic immunosuppressive which inhibits early stages of differentiation of T cells and blocks
their activation
a. bleomycin

144. Patient was diagnosed to have Huntington disease, his physician wishes to give drug, the best to give
is/are.
a. tetrobenazine
b. haloperidol
c. phenothiazine
d. all of the above
145. A pro-drug that acts on platelets:
a. heparin
b. enoxaparin
c. warfarin
d. ticlopidine
145. Simvastatinand prevastatin will
a. increase clearance of IDL and LDL
b. reduce VLDL secretion from the liver
c. decrease VLDL by stimulation of lipoprotein lipase
d. reduce LDL by an unknown mechanism

146. A weak acid that compete with uric acid in renal tubule for reabsorption

b. labetalol

a. allopurinol

c. phentolamine

b. sulfinpyrazone

d. isoproterenol

c. phenocitine
d. acetazolamide
147. An antithyroid drug that inhibits organification and thyroid hormone release

148. Which of the ff. drugs of choice in anaphylaxis associated with bronchospasm and hypotension?
a. cortisone
b. epinephrine

a. methimazole

c. isoproterenol

b. Lugols solution

d. phemytephrine

c. I131

e. terbutolone

d. propanolol
148. A partial agonist that binds progestin androgen and glucorticoid receptors used in
a. clomiphene
b. danazol
c. mifepristone
d. relaxin
149. True of proinsulin
a. an 86- amino acid single chain polypeptide
b. cleavage and cross linking result in one chain 51 peptide insulin molecule
c. can be measured by immunoassay independently of insulin
d. all of the above
146. Streptomycinc an be used against all except
a. tuberculosis
b. tularemia
c. serratia
d. brucella
147. A 88 year old man came in because of difficulty in starting to urinate PE revealed a BP 160/100 mmHg
& a slightly enlarged prostate. Which of the ff. medications is useful in treating both conditions?
a. alfuzosin

149. A 40-yr old patient presents with an acute hypertensive crisis. Which agent will act to block the
increased blood pressure by acting at sympathetic ganglia?
a. propranolol
b. trimetaphan
c. nitroglycerin
d. digoxin
e.labetalol
150. A patient was diagnosed as suffering from chronic refractory congestive heart failure with oliguria.
Which one of the ff. agents is most effective?
a. acetylcholine
b. atropine
c. dopamine
d. terbutaline
e. propranolol
151. A patient with history of bronchial asthma can be given a selective B1 adrenergic receptor blocker
a. propronolol
b. nadolol
c. timolol
d. metoprolol
e. pindolol

152. A 24 yr old female consulted because of headache palpitations and sweating BP taken
190/110mmHg.to confirm the presence of pheochromocytoma .the preferred drug would be

a.
b.
c.
d.

Piperocan
Phenoxybenzamine
Histamine
Phentolamine

153. Subsequent VMA essay done on the patient above showed elevated results medically, this patient can be
managed with the following agents except.
a. phentolamine
b. propranolol
c. atenolol
d. phenylephrine
e. clonidine
154. The adrenergic receptor most likely responsible for the cardiac stimulation observed ff. intravenous
injection of epinephrine is:

c. premature labor
d. hypertension
157. A nonselective adrenergic agent, this binds to both beta one and beta receptors and can be used in heart
block
a. epinephrine
b. ephedrine
c. isoproterenol
d. albuterol
158. Most common adverse effects of beta 2 selective adrenergic drugs
a. hypotension
b. hemolysis
c. tremors
d. nausea and vomiting
159. This side effect of inhalational steroids can be minimizes by washing the mouth after its use

a. alpha 1 adrenergic receptors

a. gingival hyperplasia

b. alpha 2 adrenergic receptor

b. oral candidiasis

c. beta 1 adrenergic receptors

c. sore throat

d. beta 2 adrenergic receptors


155. The nonselective beta adrenergic blocking agent that is also competitive antagonist at alpha
adrenoreceptors is:
a. timolol
b. nadolol
c. pindolol
d. labetalol
e. propranolol
156. Ridorine hydrochloride is used in the treatment of:
a. parkinsons disease
b. bronchial asthma

160. The clinical hallmarks of asthma include the ff. except;


a. coughing
b. shortness of breath
c. fever
d. wheezes
161. Considered as controllers in asthma
a. beclomethesone
b. salbutamol
c. ipratropium bromide
d. epinephrine
162. True of glucocorticoids, except:

a. decrease formation of cytokines


b. inhibit the generation of PGE2 and PGI 2

d. nifedipine
166. Cromolyn sodium as its major action

c. inhibit the production of spasmogens


d. reverse the initial bronchoconstriction in asthma
163. This mucolytic agent breaks disulfide bonds and is useful as an antidote for paracetamol
a. carbocystiene
b. acetylcysteine
c. bromhexine
164. Known as chemoattractants, action is on the later phase of asthma:
a. LTB4
b. LTC4
c. LTD4
d. all of the above
163. True of salbutamol, except;

a. block of calcium channels in lymphocytes


b.

block of mediator release from most cells

c. block of phosphodiesterase in most cells and basophils


d. smooth muscle relaxation in the bronchi
169. A 16 yr old patient is in the emergency room. She has a heart rate 130 per min. 0f 40/min and an
estimated 1-second forced expiratory volume less than 10% of end rates are audible without a stethoscope.
Drugs that can dilate bronchioles during an acute asthmatic attack include all of except;
a. epinephrine
b. terbutaline
c. nedocromil
d. theophylline
169. After successful treatment of the acute attack. This patient was on follow-up treatment of her asthma.
Succesful strategies currently in use are the following except.

a. given by oral and inhalational route

a. avoidance of antigen exposure

b. maximum effect is within 30 minutes

b. blockade of histamine receptor

c. duration is 4-6 hours

c. blockade of leukotriene

d. blocks muscarinic receptors

d. inhibition of phospholipase

164. These drug cause relatively little respiratory depression unless combine with alcohol
a. opiates

170. True of Cromolyn sodium


a. in as aerosol that effectively inhibit both antigen and exercise induced asthma

b. benzodiazepine

b. in chronic use slightly reduces the overall level of bronchial reactivity

c. barbiturates

c. efective on airway smooth muscle tone and are effective in reversing bronchospasm

d. anesthetics

d. only a and b

165. Preliminary studies in animals suggest that airway smooth muscle like that in the vascular is effectively
relaxed by this agent. Possibly useful in pulmonary hypertension;
a. nutric oxide
b. cromakallin
c. lgE antibodies

180. One effect that theophylline, nitroglycerin, isoproterenol have in common is


a. direct stimulation of cardiac contractile force
b. tachycardia
c. increased gastric acid
d. postural hypotension

182. Theophylline acts by;


a. inhibiting phosphodiesterase
b. inhibiting cylic AMP

b. sulfur dioxide
c. nitrogen oxides
188. This is a colorless, irritant gas, generated primarily by the combustion of fossil fuels;

c. inhibiting beta receptor

a. carbon moxide

d. inhibiting muscarine

b. sulfur dioxide

183. The xanthine drugs produce this effects;


a. bronchodilation
b. decrease gastric and digestive enzymes

c. nitrogen oxides
189. This is a brownish, irritant gas, sometimes associated with fires:
a. carbon moxide

c. CNS depression

b. sulfur dioxide

d. all of the above

c. nitrogen oxide

183. Beta adrenergic agents cause;


a. stimulation of adenyl cyclase
b. inhibition of phosphodiesterase
c. antagonize adenyl cyclase
d. all of the above
184. Plasma concentration of theophylline is increased when given with
a. cimetidine
b. rifampin
c. phenytoin
185. To relieve the bronchospasm in bronchial asthma, you may give the ff., except
a. metoprolol
b. ipratropium bromide
c. isoproterenol
186. True of ipratropium bromide except
a. binds to all muscarinic receptor
b. inhibits acetylcholine mediated bronchospasm
187. This is a colorless, tasteless, odorless and non- irritating gas, a byproduct of incomplete
a. carbon monoxide

189. This is a bluish irritant gas that occurs normally in the earths atmosphere, when absorbent of ultraviolet
light;
a. carbon monoxide
b. sulfur dioxide
c. nitrogen oxides
d. ozone
190. Asthma is often exacerbated in patients exposed to this reducing agent whenair as low as 1-2 ppm. It
is formed mainly form combustion agent of fossil fuels;
a. sulfur dioxide
b. rotenone
c. carbon monoxide
101. Acute exposure to this alphatic hydrocarbon solvent causes CNS depression, this has led to impairment
of memory and peripheral neuropathy:
a. tetrachloroethylene
b. toluene
c. paraqual
102. This compound is a potential environmental hazard that is formed as a contaminant in the manufacture
of herbicides;
a. DDT

b. dioxin
c. aldicarb
d. benzene
193. Bone marrow cell in early stage of their development appear to be most sensitive to this drug and can
cause pancytoponia and aplastic anemia
a. carbon monoxide
b. carbon dioxide
c. benzene
194. Decrease incidence of EPS side effects;
a. chlorpromazine
b. triflupromazine
c. fluphenazine
d. haloperidol
e. thioridazine
195. Increase EPS but decrease tendency to produce sedation of autonomic side effects;
a. fluphenazine
b. chlorpromazine
c. thioridazine
d. mesoridazine
e. piperocetazine
True of the pharmacokinetics of antipsychotic drugs;

a.
b.
c.
d.

Highly lipid-soluble and protein bound


Longer clinical duration than the half life
Prolonged half life
A and b

Antagonism of dopamine in the nigrostriatal system

a.
b.
c.
d.

Parkinsonism
Galactorrhea
Amenorrhea
Hyperprolactinemia

e. Psychosis
Blocking of dopamines tonic inhibitory effect in prolactin release in the pituitary gland causes

a.
b.
c.
d.
e.

Parkinsonism
Galactorrhea
Amenorrhea
Hyperprolactinemia
Psychosis

Blockade of the dopamine in the mesolimbic and mesofrontal system;

a.
b.
c.
d.
e.

Antipsychotic effect
Depression
Hyperprolactinemia
Amenorrhea
Galactorrhea

Effects on the medullary periventricular pathway

a. Eating disorder
b. Infertility
True about clindamycin except

a.
b.
c.
d.

Excellent oral absorption


Associated with antibiotic associated enterocolitis
Poor bone penetration
Prophylaxis of endocarditis in patients with valvular hearth disease for
surgical procedures
e. Blocks translocation reaction
Route of elimination of streptogramins

a.
b.
c.
d.

Fecal
20% urine
Bile
A and b

Its unique binding site results in no cross resistance with other drug classes

a. Clindamycin
b. Erythromycin
c. Linezolid

d. Synercid
e. Lyncomycin
Contraindications for fluoroquinolone use except

a.
b.
c.
d.
e.

Elderly
A 30yr old patient with complicated urinary tract infection
Patients who are in their prepubertal stage
Patients taking theophyline
A and b

The following are systemic acting quinolone except

a. Ciprofloxacin
b. Nalidixic acid
c. Gatfloxacin
Inhibits the reabsorption of uric acid by a weak acid carrier mechanism at the part of nephron;

a.
b.
c.
d.

Probenecid
Coichicine
Sulfinpyrazone
Allopurinol

Indications for allopurinol use except;

a.
b.
c.
d.
e.

Chronic tophaceous gout


Allergic reactions to uricosuric agents
Recurrent renal stones
Normal serum urate
Renal functional impairment

True about Aspirin;

a.
b.
c.
d.
e.

Poor inhibitor of cyclooxygenase in peripheral tissues


Contraindicated in children
Crosses BBB and placental barrier by active transport
No anti-inflammatory effect
All of the above

True about the toxic effects of Aspirin except;

a. Reye syndrome
b. Salt and water retention

c. Prolongation of bleeding time


d. Antidote is N-acetylcysteine
e. Allergic reaction
Therapeutic indications of diclofenac;

a.
b.
c.
d.
e.

Rheumatoid arthritis
Dysmenorrheal
Acute MS injury
Postoperative pain
All of the above

Chemotherapeutic agents classified as plant alkaloid except;

a.
b.
c.
d.

Paclitaxel
Teniposide
Vincristine
NOTA

Chemotherapeutic agent classi fied as podophyflin alkaloid except

a.
b.
c.
d.

Teniposide
Vp-16
Vincristine
Etoposide

Chemotherapeutic agent classified as alkyiating agent except;

a. Busulfan
b. Plicamycin
c. Cyclophosphamide
Site of alkylation within DNA of alkytating agents is

a. N7 of guanine
b. N2 of adenine
c. N2 of cytosine
Mechanism of resistance to alkytating agents involves;

a.
b.
c.
d.

Increased capability to repair DNA lesions


Decreased permeability of cell to the alkylating drug
Increased production of glutathione
All of the above

Alkylating agent mostly used for CML

a. Busulfan
b. Cyclophosphamide
c. Dactinomycin
Function by cross linking through alkylation of DNA

a. Nitrosoureas
b. Antibiotics
c. Antimetabolites
Cancer chemotherapeutic agent classified as plant alkaloid

a. Vincristine
b. Dactinomycin
c. Bleomycin
Intermediate acting tetracyclines;

a.
b.
c.
d.
e.

Demaclocycline
Methecycline
Oxytetracycline
Chlortetracycline
A&B

Toxic effects of tetracycline except;

a.
b.
c.
d.
e.

Renal toxicity
Temporary discoloration of teeth in children
GI distress
Inhibits bone growth
Teratogenic

Toxic effects of chloramphenicol except;

a.
b.
c.
d.
e.

GIT disturbances
Vaginal candidiasis
Reversible aplastic anemia (irreversible)
Dose dependent bone marrow inhibition
Gray baby syndrome

True about the macrolides

a.
b.
c.
d.
e.

Binds to the P sites of the 30 S bacterial ribosomal subunit


Erythromycin is the prototype agent
Not active against Chlamydia
Only inhibitory in action
Consist of a single membered lactone ring

Oral preparations of erythromycin;

a. Lactobionate
b. Estolate salts
c. Glucaptate
Which of the ff. is used for prophylaxis treatment of DVT;

a. Heparin
b. Warparin
c. Aspirin
Drug that reverses quickly the effect of oral anti-coagulant drugs;

a. Dicoumarol
b. Vitamin K
c. Protamine sulfate
The ff. drugs reduces platelet aggregation by inhibiting the ADP pathway

a. Clopidogrel
b. Abciximab
c. Aspirin
Account for the most anti-coagulant effect of heparin except

a.
b.
c.
d.

Inhibition of thrombin
Inhibition of factor VII
Inhibition of Xa
All of the above

The presence of tissue thromboplastin is needed to activate this pathway

a.
b.
c.
d.

Intrinsic pathway
Common pathway
Extrinsic pathway
All of the above

Fibrinolytic activators;

a.
b.
c.
d.

Urokinase
Plasmin
Ticlopidine
Vit.k

Which of the ff. clotting factors is greatly affected by heparin except;

a. Thrombin
b. IXa
c. XIIa
Clinical effects of heparin is best monitored by;

a.
b.
c.
d.
e.

Prothrombin time
Complete blood count
Bleeding time
Platelet count
Partial thromboplastin time

The following are available Vit B12 for therapeutic, EXCEPT

a.
b.
c.
d.
e.

Adenosylcobalamin
Methylcobalamin (pero ito yung sagot sa samplex)
Cyanocobalamin
Hydroxocobalamin
None of the above

A patient comes into a emergency room is pulse of 140 & a bp of 190/120 he is .complaining of insomnia.
Which of the ff. drugs could have caused these symptoms?

a.
b.
c.
d.

Metaramiol
Ephedrine
Phenylephrine
Amphetamine

All of the ff. are true of trimethaphan except

a.
b.
c.
d.

Increases BP
Short-acting
Binds to the cholinergic nicotinic receptor
Competes for acetylcholine receptor sites

All of the ff. are true concerning scopolamine except;

a.
b.
c.
d.

Plant alkaloid
Effects similar to atropine
Used in treatment of motion sickness
Competes with adrenergic receptors

The toxicity of irreversible cholinesterase inhibitors is treated with

a. Atropine
b. Atropine and pralidoxime
c. Praidoxime
Direct acting cholinomimetics that bind and activate both the muscarinic and nicotinic receptors

a.
b.
c.
d.

Physostigmine
Edrophonium
Metacholine
Malathione

True of physostigmine

a.
b.
c.
d.

Contains a tertiary ammonium group


Water soluble
Does not enter the CNS
Short acting

Which of the ff. antimuscarinic drugs is used by inhalation in the treatment of bronchoconstriction

a. Dicyclomine HCI
b. Methscopotamine bromide
c. Ipratroium bromide
The cholinesterase inhibitor that is used in the diagnosis of myasthenia gravis is

a. Edrophonium chloride
b. Ambenenium
c. Malathion
Atropine and scopolamine will block all the effects of acetylcholine listed below except

a. Bradycardia
b. Salivary secretion
c. Bronchoconstriction

d. Skeletal muscle???
The efferent nerves of this system supplies only the skeletal muscles

a.
b.
c.
d.

Somatic nervous system


Autonomics nervous system
Central nervous system
All of the above

Which of the ff. statements is true about angiotensin receptor blockers

a.
b.
c.
d.
e.

Inhibits angiotensin action more completely than ACE inhibitors


No effect on bradykinin metabolism
Have the same side effect as ACE inhibitors except for cough and
All of the above
A and b only

Drug that have been shown to reduce morbidity and mortality in hypertension

a.
b.
c.
d.
e.

Diuretics
Beta blockers
Alpha blockers
All of the above
A and B only

Antihypertensive agent that is useful in patients with angina;

a.
b.
c.
d.

Beta blocker
Calcium channel blocker
Hydralazine
A and b only

Antihypertensive agents that is useful in patients who also have congestion

a.
b.
c.
d.

Diuretics
ACE inhibitors
Alpha blockers
A and b only

Side effect related to use of ACE inhibitors include which of the ff.

a. Acute renal failure


b. Cough and angioedema
c. Hyperkalemia

d. A and b only
e. All of the above
Vasodilator therapy leads to which of the ff. compensatory response

a.
b.
c.
d.
e.

Increase renin release


Tachycardia
Orthostatic hypotension
All of the above
A and B only

Drugs with positive inotropic action includes which of the ff.

a.
b.
c.
d.
e.

Captopril
Labetalol
Milrinone
All of the above
A and b only

Which of the ff. statements is true about digitalis glycosides

a.
b.
c.
d.
e.

Inhibits Na-KATPase
Increases calcium concentration leading to grater systolic contraction
Causes increase in intracellular Na concentration which leads to.
All of the above
A and b only

Edema in patients with congestive heart failure is due to which mechanism

a. Decrease in renal blood flow activating the RAAS system causing


retention of salt
b. Increase in venous pressure causes an increase in capillary filling
pressure
c. Decrease in blood pressure activating the adrenergic system leading to
increase arteriolar resistance and systemic blood pressure
d. All of the above
e. A and b only
Beneficial effects of digitalis include which of the ff.;

a. Increase in heart rate


b. Increase in cardiac output

c. Increase in production..
d. All of the above

You might also like